Está en la página 1de 94

Borrador del Módulo de Matemática II

BORRADOR
DEL
MÓDULO DE
MATEMÁTICA II

1
Jorge Tuapanta
Borrador del Módulo de Matemática II

CÁLCULO DIFERENCIAL EN VARIAS VARIABLES

1.1. FUNCIONES VECTORIALES DE VARIABLE REAL


Definición.- Se llama función vectorial de variable real f : A  R  R n si a cada
n
elemento de A se le hace corresponder mediante f un elemento único de R ,

tal que: f (t )   f1 (t ), f 2 (t ), f 3 (t ),... f n (t )  con f1 (t ), f 2 (t ), f 3 (t ),... f n (t ) funciones

reales de variable real.


Definición.- Sean f1 (t ), f 2 (t ), f 3 (t ) funciones reales definidas de A en R,

entonces t  D f1  D f 2  D f3 , existe un vector definido por:

f (t )  f1 (t )i  f 2 (t ) j  f 3 (t )k , al vector f (t ) se le llama función vectorial de


variable real, al cual denotaremos por:

f : A  R  R 3 tal que
t  f (t )   f1 (t ), f 2 (t ), f 3 (t ) 
donde f1 (t ), f 2 (t ), f 3 (t ) son las componentes de la función vectorial.

El dominio de la función vectorial f (t )  f1 (t )i  f 2 (t ) j  f 3 (t )k lo definiremos

como D f  D f1  D f 2  D f3 , es decir, el dominio de f (t ) depende del dominio

de las funciones componentes.

El rango de la función f (t ) se define por : R f  R (campo vectorial).


3

EJEMPLO 1

La función f (t )  (1,4,5)  t (3,1,5)  (1  3t ,4  t ,5  5t ) que representa una recta


que pasa por (1,4,5) y tiene vector director (-3,1,5), tiene como dominio todo R; el
3
rango es una recta en R .

EJEMPLO 2

1
Jorge Tuapanta
Borrador del Módulo de Matemática II

La función f (t )  (cos t , sent ), con t  [0,2 ] tiene como rango una circunferencia
2

en R , es decir: R f  ( x, y)  R / x  y  1
2 2 2

EJEMPLO 3

La función f (t )  (3t , t ) tiene como dominio todo R , y rango una parábola en


2

 x2 
R , es decir: R f  ( x, y)  R / y  
2 2

 9
TAREA: ESPIMOZA RAMOS, Análisis matemático III. Pág. 85 todos

1.2. LÍMITE DE UNA FUNCIÓN VECTORIAL

Definición.- Sea f (t ) una función vectorial y t 0 un punto de acumulación del

dominio de f, se dice que el vector b es el límite de f (t ) cuando t se acerca a t 0

y se denota como lim f (t )  b , si solo si,


t t0

  0   0 / 0  t - t 0    f (t )  b   , es decir:

lim f (t )  b    0   0 / 0  t - t 0    f (t )  b  
t t0

TEOREMA: Sea f : A  R  R , una función vectorial de variable real, entonces


n

lim f (t )  L si solo si lim f i (t )  Li , i  1,2,3,..n donde L  L1 , L2 , L3 ,...Ln 


t t0 t t 0

NOTA: El teorema establece, que límite de una función vectorial es igual al vector

cuyas componentes son los límites de las respectivas componentes de f (t ) , es

decir: lim f (t )   lim f1 (t ), lim f 2 (t ), lim f 3 (t ),... lim f n (t ) 


t t0  t t0 t t0 t t0 t t0 
EJEMPLO 1

 1

 1 t t 3  t 1 t  1 t 
Hallar el lim f (t ) si f (t )     , , 
t0 1 t  t t 
 
 1

  1 t t3  t 1 t  1 t 
lim f (t )   lim   , lim , lim   (e 1 ,1,1)
t 0 1  t
t 0
   t 0 t t  0 t 
 

2
Borrador del Módulo de Matemática II

EJEMPLO 2

t 2  2t  3 t 2  5t  6
Hallar el lim f (t ) si f (t )  i j
t3 t 3 t 3
t 2  2t  3 t 2  5t  6
lim f (t )  lim i  lim j  4i  j
t 3 t 3 t 3 t 3 t 3
EJEMPLO 3
sen t cos t - 1
Hallar el lim f (t ) si f (t )  i j  et k
2

t0 t 2t
sen t cos t - 1
lim f (t )  lim i  lim j  lim e t k  (1,0,1)
2

t 0 t 0 t t 0 2t t 0

PROPIEDADES DE LÍMITES

Sean f (t ) y g (t ) funciones vectoriales tales que lim f (t )  L y lim g (t )  F


t t0 t t0

y t 0 un punto de acumulación de D f  Dg , entonces:


t t0

1. lim f (t )  g (t )  lim f (t )  lim g (t )  L  F
t t0 t t0

2. lim f (t )  g (t )  lim f (t )  lim g (t )  L  F


t t0 t t0 t t0

3. lim f (t ) x g (t )  lim f (t ) x lim g (t )  Lx F


t t0 t t0 t t0

TAREA: ESPIMOZA RAMOS, Análisis matemático III. Pág. 130-131 del 3 al 18

1.3. CONTINUIDAD DE FUNCIONES VECTORIALES

Definición.- La función f (t ) es continua en t 0 si: 1) f (t 0 ) existe, es decir

t 0  D f 2) lim f (t ) existe y 3) lim f (t )  f (t 0 )


tt 0 t t 0

TEOREMA.- f (t )  f1 (t )i  f 2 (t ) j  f 3 (t )k es continua en t 0 , si y solo si,

f i , i  1,2,3 es continua en t 0 .
EJEMPLO 1

La función f : R  R3 definida por f (t )  (t 3  3t  5, t 2  t  2, t  5) es


continua, porque sus componentes son funciones polinómicas, y por tanto son
continuas.

3
Borrador del Módulo de Matemática II

EJEMPLO 2

 2 sen t 
 t , t ,  si t  0
La función f : R  R 3 definida por f (t )   t  , es discontinua
(0,0,0) si t  0

en t=0, ya que:

 sen t 
lim f (t )  lim t , t 2 ,   (0,0,1)  (0,0,0)  f (0)
t 0 t 0
 t 

Como el lim f (t )  f (0) entonces f (t ) es discontinua en t=0.


t 0

TEOREMA.- Si  es continua en t 0 y f (t ) es continua en  (t 0 ) entonces

f   es continua en t 0 .

PROPIEDADES DE LA CONTINUIDAD

Sean f , g : R  R funciones vectoriales continuas en t 0  R , entonces:


3

1. f  g es continua en t 0

2. f  g es continua en t 0

3. f (t ) xg(t ) es continua en t 0

4. ( f )(t ) es continua en t 0 ,   R

NOTA: La función vectorial f : R  R es continua en un conjunto A  Df si la


3

función f (t ) es continua en cada uno de los puntos de A.

1.4. DERIVADA DE UNA FUNCIÓN VECTORIAL

La derivada de una función vectorial de variable real f : [a, b]  R 3 , tal que

f (t  h)  f (t )
f (t )  f1 (t )i  f 2 (t ) j  f 3 (t )k , está definida por f (t )  lim
t 0 h

4
Borrador del Módulo de Matemática II

siempre y cuando exista este límite, y lo denotaremos por:


d
f (t )  Dt f (t )  f (t ) .
dt

TEOREMA.- Si f (t )  f1 (t )i  f 2 (t ) j  f 3 (t )k entonces

     
f (t )  f1 (t )i  f 2 (t ) j  f 3 (t )k siempre que f1 (t ), f 2 (t ), f 3 (t ) existan.

Definición.- La función vectorial f : [a, b]  R , se dice que es diferenciable en


3

el intervalo {a,b]; si f (t ) existe t [a, b] .

PROPIEDADES DE LA DIFERENCIACIÓN

Consideremos las funciones vectoriales f (t ) y g (t ) , y   R entonces:

   
1. D t  f (t )   D t f (t )

2. D  f (t )  g (t )  f (t ) D g (t )  g (t ) D  f (t ) 
t t t

3. D  f (t ) x g (t )  f (t ) x D g (t )  g (t ) xD  f (t ) 
t t t

VECTOR VELOCIDAD, RECTA TANGENTE, VECTOR ACELERACIÓN Y


RAPIDEZ

Sean f : [a, b]  R una función vectorial diferenciable, al vector no nulo f (t ) se


3

denomina vector velocidad ( vector tangente) de la curva en el punto f (t )  R , y


3

se denota por v(t )  f (t ) .

El vector velocidad f (t 0 )  0 , determina una recta tangente L a la curva en el

punto f (t 0 ) 
dada por: Lt  f (t 0 )  t f (t ) / t  R . 
Al vector aceleración se lo define como a(t )  v (t )  f (t )
La rapidez de una partícula se lo define como la magnitud del vector velocidad, es

        
2 2 2
decir: v(t )  f (t )   f1 (t )    f 2 (t )    f 3 (t ) 
     
EJEMPLO 1

Sea f : R  R , dado por f (t )  (3cos t,3sen t).


2

5
Borrador del Módulo de Matemática II

El recorrido de la función es una circunferencia de radio 3: x  y  9 t  R


2 2

El vector velocidad de f (t ) es: v(t )  f (t )  (-3sen t, 3 cos t)

El vector aceleración es: a(t )  f (t )  (-3 cos t, - 3 sen t)


EJEMPLO 2
5

Sea f : R  R , dado por f (t )  (t , (t  3) , sen 7t) .


3 2 5

 5 32 
El vector velocidad de f (t ) es: v(t )  f (t )   t ,5(t  3) ,7 cos 7t 
 4

2 
 15 12 
El vector aceleración es: a(t )  f (t )   t ,20(t  3) ,49 sen 7t 
3

4 
EJEMPLO 3
Una partícula se mueve describiendo una hélice circular

f (t )  a cos wt i  a sen wt j  b.wt k a, b, w  0 , para cada instante t, hallar:


a) La velocidad
b) La aceleración
c) La magnitud de la aceleración
d) El ángulo que forma los vectores aceleración y velocidad

v a
Utilice para hallar el ángulo    (v, a ) la fórmula cos  
v a

TEOREMA.- Si f (t ) es una función vectorial diferenciable en un intervalo B y

f (t ) es un vector diferente de cero de magnitud constante y dirección variable

t  B , entonces f (t ) y Dt f (t ) son ortogonales.


EJEMPLO 1

La función vectorial f (t )  (cos t, sen t) t  R, f (t )  1 .

f (t )  (sen t, cos t) t  R

f (t )  f (t )  (cos t, sen t)  (-sen t, cos t)  -cos t. sen t  en t. cos t  0

Como f (t )  f (t )  0 entonces f (t ) y f (t ) son ortogonales

6
Borrador del Módulo de Matemática II

NOTA: La derivada de orden superior de las funciones vectoriales están definidas


en relación con las derivadas de orden superior de las funciones reales.

Si f (t )  f1 (t )i  f 2 (t ) j  f 3 (t )k , entonces:

  
La primera derivada es f (t )  f1 (t )i  f 2 (t ) j  f 3 (t )k

  
La segunda derivada f (t )  f1 (t )i  f 2 (t ) j  f 3 (t )k
( n)
(t )  f1 (t )i  f 2 (t ) j  f 3 (t )k
( n) ( n) ( n)
La n-ésima derivada es f
EJEMPLO 2
(n) 1 1
Encontrar f (t ) de f (t )  i j
2t  3 1  t
 2.1 1
f (t )  i j
(2t  3) 2
(1  t ) 2

2 2.1.2 1.2
f (t )  i  j
(2t  3) 3 (1  t ) 3

 2 3.1.2.3 1.2.3
f (t )  i j
(2t  3) 3
(1  t ) 4
.
.
.
( n) (2) n .1.2.3...n 1.2.3...n
f (t )  i  j
(2t  3) n 1 (1  t ) n1

(n) (2) n .n ! n!
Por tanto f (t )  i  j
(2t  3) n 1 (1  t ) n1

TEOREMA.- Si g : A  R es una función diferenciable en A y f : R  R 3 es


una función diferenciable sobre un intervalo que contiene a g ( A)  g (t ) / t  A ,

entonces f  g es diferenciable sobre A y


d
dt
 
f ( g (t ))  f ( g (t )).g (t ) t  A .

EJEMPLO 1

7
Borrador del Módulo de Matemática II

SI g (t )  e
 t
t  [0, [ y f (t )  (t , t 2 ,
t3
3
) t ] 0, [ . Hallar
d
dt

f ( g (t )) . 
  
f (t )  (1, 2t , t 2 )  f ( g (t ))  1, 2 g (t ), g 2 (t )  1, 2e  t , e 2  t 
g (t )   e  t .Entonces:
d
dt
 
f ( g (t ))  f ( g (t )).g (t )  1, 2e  t , e 2  t   e  t    e  t 1, 2e  t , e 2  t 
TAREA: ESPIMOZA RAMOS, Análisis matemático III. Pág. 140 del 29 al 35

1.5. FUNCIONES REALES DE VARIABLE VECTORIAL


Definición.- Se llama función real de variable vectorial f : A  R  B  R , si
n

a cada elemento x  x1 , x2 , x3 ,...xn   A de se le hace corresponder un

elemento único f ( x)  B .

El valor de la función f se denota por z  f x1 , x2 , x3 ,...xn  donde las variables

x1 , x2 , x3 ,...xn son las variables independientes, y z la variable dependiente.


DOMINIO Y RECCORRIDO DE LA FUNCIÓN REAL DE VARIABLE
VECTORIAL

Consideremos la función f : R n  R , el dominio de la función, que lo

denotamos como D f , lo definimos por:


D f  x  x1 , x2 , x3 ,...xn   R n / z  R  z  f x1 , x2 , x3 ,...xn  
El rango de la función, que denotamos por R f , lo definimos como:


R f  z  R /  x  x1 , x2 , x3 ,...xn   R n  z  f(x) 
Consideremos una función real de dos variables reales, es decir, una función f
que asigna a cada pareja ordenada (x,y) en algún conjunto A del plano uno y sólo
un número real f(x,y) de B. Por ejemplo:
f ( x, y)  x  y 2

g ( x, y)  x x  y

h( x, y)  log( x 2  y)

8
Borrador del Módulo de Matemática II

Para f ( x, y)  x  y 2 el dominio es: D f  ( x, y) / x, y  R, de g ( x, y)  x x  y su

dominio es Dg  ( x, y) / x  y  0 y de h( x, y)  log( x 2  y) el dominio es

 
Dh  ( x, y) / x 2  y  0 que representa la parte del plano por encima de y   x 2 . Al

conjunto de valores f (x,y) correspondiente a dicho dominio se llama recorrido de f.

Definición.- Consideremos dos funciones f , g : R n  R con D f y Dg


respectivamente sus dominios, entonces:

1. ( f  g )( x)  f ( x)  g ( x), x  D f  g  D f  Dg

2. ( f .g )( x)  f ( x).g ( x), x  D f . g  D f  Dg

3. 
f
( x) 
f ( x)
, x  D f / g  D f  D g  x / g ( x )  0  
g g ( x)

Definición.- Consideremos f : Rn  R y g:RR con Df y Dg


respectivamente sus dominios, entonces definimos la función compuesta como:

 
( g  f )( x)  g f ( x)  g  f x1 , x2 , x3 ,...xn  , donde


Dg  f  x  D f / f ( x )  R f  Dg . 
EJEMPLO

Dado g ( x)  ar cos x y f ( x, y)  x 2  y 2  16 , hallar gof y su dominio

Hallemos sus dominios: Dg  [1, 1 ] y D f  ( x, y)  R / x  y  16  0  2 2 2



  
Dg f  ( x, y)  R 2 / f ( x, y)  Dg  ( x, y)  R 2 /  1  x 2  y 2  16  1 
Dg f  ( x, y)  R 2
/ 0  x 2  y 2  16  1 
Dg f  ( x, y)  R 2
/ 16  x 2  y 2  17 
( g  f )( x, y)  g  f ( x, y)  g x 2

 y 2  16  ar cos x 2  y 2  16
TAREA: ESPIMOZA RAMOS, Análisis matemático III. Pág. 278-279 los pares

Definición.- La gráfica de una función de n variables ( x1 , x 2 , x3 ,...x n ) es el conjunto

de puntos ( x1 , x 2 , x3 ,...xn , z )  R n1 que satisfacen f ( x1 , x 2 , x3 ,...x n ) con

( x1 , x 2 , x3 ,...x n ) en el dominio de f.

9
Borrador del Módulo de Matemática II

Para n = 1. La gráfica de una función f : R  R generan los puntos (x, y) tales


que y  f (x) , y se representa como una curva en R2.
Por ejemplo f ( x)  x senx
y

        









Para n = 2. La gráfica de una función f : R 2  R la generan los puntos (x, y, z)


tales que z  f ( x, y) y se puede representar como una superficie en R3. Por

sen( x 2  y 2 )
ejemplo z  f ( x, y ) 
x2  y2
z

LÍMITES
Vamos a tratar el concepto de límite de forma intuitiva. Para una función f de una
variable, decimos que lim f ( x)  L si existen los límites por la derecha y por la
xa

izquierda de x = a, y además coinciden con el valor de L.


Para una función f de dos variables, decimos que lim f ( x, y)  L si existen los
( x , y ) ( a ,b )

límites por todos los caminos posibles hacia el punto (a, b), y además coinciden
con el valor de L.

10
Borrador del Módulo de Matemática II

Definición.- Que lim f ( x, y)  L significa que para cada   0 (sin importar


( x , y ) ( a ,b )

que tan pequeño sea) existe un   0 correspondiente de modo que


f ( x, y)  L   , siempre que 0  ( x, y)  (a, b)   .

Note algunos aspectos de la definición:


1. La trayectoria de acercamiento a (a,b) no es importante. Esto significa que si
distintas trayectoria de acercamiento conducen a distintos valores L, entonces
el límite no existe.
2. El comportamiento de f(x,y) en (a,b) no es importante; la función ni siquiera
tiene que estar definida en (a,b). Esto es consecuencia de la restricción
0  ( x, y)  (a, b) .

3. La definición se puede extender para funciones de n variables, sólo se tiene


que reemplazar (x,y) por ( x1 , x 2 , x3 ,...x n ) y (a,b) por (a1 , a 2 , a3 ,...a n ) .
PROPIEDADES DE LÍMITES
Sean f y g funciones tales lim f ( x, y)  L y lim g ( x, y)  F entonces
( x , y ) ( a ,b ) ( x , y ) ( a , b )

1. lim ( f  g )( x, y)  lim f ( x, y)  lim g ( x, y)  L  F


( x , y ) ( a ,b ) ( x , y ) ( a ,b ) ( x , y ) ( a ,b )

2. lim ( f  g )( x, y)  lim f ( x, y)  lim g ( x, y)  L  F


( x , y ) ( a ,b ) ( x , y ) ( a ,b ) ( x , y ) ( a ,b )

f lim f ( x, y )
( x , y ) ( a , b ) L
3. lim  ( x, y )  ) , F0
( x , y )  ( a , b ) g 
  lim
( x , y ) ( a ,b )
g ( x, y ) F

lim f ( x, y )
4. lim e f ( x, y )  e ( x , y ) ( a ,b )
 eL
( x , y ) ( a , b )

5. lim log f ( x, y)  log lim f ( x, y)   log L, L  0


( x , y ) ( a , b )  ( x , y ) ( a , b ) 

11
Borrador del Módulo de Matemática II

TEOREMA.- Si la función f : R 2  R , tiene límites diferentes cuando (x,y) se

aproxima a ( x 0 , y 0 ) a través de dos conjuntos diferentes que tienden a ( x 0 , y 0 )

como punto de acumulación entonces lim f ( x, y) no existe.


( x , y ) ( x 0 , y 0 )

e x y
EJEMPLO 1: Hallar el límite de lim
( x , y ) ( 0 , 0 ) x2  y2  3
Utilizando la propiedad 3 tenemos que:
lim e x y
e x y ( x , y ) ( 0 , 0 ) e 00 1
lim   
( x , y ) ( 0 , 0 ) x2  y2  3 lim x2  y2  3 003 3
( x , y ) ( 0 , 0 )

x2 y2
EJEMPLO 2: Hallar el límite de lim
( x , y ) ( 0 , 0 ) x 2  y 2

0
Al utilizar las propiedades nos queda que es una indeterminación.
0
Procedamos de la siguiente manera para determinar si existe el límite:
Utilizando el teorema anterior, consideremos dos caminos diferentes que tengan a
(0,0) como punto de convergencia. Sean

S  ( x, y)  R 2 / y  x  
y T  ( x, y)  R 2 / y  2 x 
x2 y2 x4 x2
El límite a través del camino S es: lim  lim  lim 0
( x , y ) ( 0 , 0 ) x 2  y 2 ( x , y ) ( 0 , 0 ) 2 x 2 ( x , y ) ( 0 , 0 ) 2

x2 y2 4x 4 4x 2
El límite a través del camino T es: lim  lim  lim 0
( x , y ) ( 0 , 0 ) x 2  y 2 ( x , y ) ( 0 , 0 ) 5 x 2 ( x , y ) ( 0 , 0 ) 5

Como los límites por dos caminos son iguales podemos concluir que
x2 y2
lim 0.
( x , y ) ( 0 , 0 ) x2  y2
CONTINUIDAD DE UNA FUNCIÓN DE VARIAS VARIABLES
DEFINICIÓN.- La función f : A  R 2  R es continua en el punto ( x 0 , y 0 )  D f si

se cumple que:
1. f(x,y) esta bien definida en ( x 0 , y 0 )  D f

2. Existe lim f ( x, y)
( x , y ) ( x 0 , y 0 )

3. lim f ( x, y)  f ( x0 , y 0 )
( x , y ) ( x0 , y 0 )

12
Borrador del Módulo de Matemática II

NOTA: La función f : A  R2  R es continua en la región abierta A si es


continua en todo punto de A.
PROPIEDADES DE LAS FUNCIONES CONTINUAS
Sea k constante, f y g funciones continuas en ( x 0 , y 0 ) , entonces las siguientes

funciones son continuas en ( x 0 , y 0 ) : 1.

f
Kf 2. f.g 3. f g 4. , con g(x 0 , y 0 )  0
g
EJEMPLO
Determinar si la función f es continua en (0,0).
 xy
 , (x, y)  (0,0)
f(x, y)   x 2  y 2
0 , (x, y)  (0,0)

Veamos si cumplen las tres condiciones de continuidad:
1 f(0,0)=0, f está definida en (0,0)  D f

2 Hallemos el lim f ( x, y)
( x , y ) ( 0 , 0 )

Calculemos el límite por dos caminos:



S  ( x, y)  R 2 / y  2 x  
y T  ( x, y)  R 2 / y  3x 
xy 2x 2 2 2
El límite a través del camino S es: lim  lim  lim 
( x , y ) ( 0 , 0 ) x 2  y 2 ( x , y ) ( 0 , 0 ) 5 x 2 ( x , y ) ( 0 , 0 ) 5 5

xy 3x 2 3 3
El límite a través del camino T es: lim  lim  lim 
( x , y ) ( 0 , 0 ) x  y
2 2 ( x , y ) ( 0, 0 ) 10 x 2 ( x , y ) ( 0, 0 ) 10 10
Como los límites por dos caminos son diferentes podemos concluir que
xy
lim no existe.
( x , y ) ( 0 , 0 ) x  y 2
2

Por tanto f(x,y) es discontinua en (0,0).


TAREA: ESPIMOZA RAMOS, Análisis matemático III. Pág. 322-324 los pares

1.6. DERIVADAS PARCIALES


Si deseamos saber cómo varía la función si variamos una sola variable
independiente manteniendo constantes las demás, o bien cómo varía la función si
nos movemos en una dirección, extenderemos el concepto de derivada a
funciones de varias variables.

13
Borrador del Módulo de Matemática II

DEFINICIÓN. - Sea f : R 2  R una función de dos variables. Diremos que la

derivada parcial de f con respecto de x en el punto ( x, y) , y se denota f x ( x, y) ,

es la función:
f ( x  h, y)  f ( x, y)
f x ( x, y)  lim
h0 h
siempre que exista el límite.
DEFINICIÓN. - Sea f : R 2  R una función de dos variables. Diremos que la
derivada parcial de f con respecto de y en el punto ( x, y) , y se denota f y ( x, y)

, es la función:
f ( x, y  h)  f ( x, y)
f y ( x, y)  lim
h0 h
siempre que exista el límite.
f z f z
NOTACIÓN: f x ( x, y )  f x   y f y ( x, y )  f y  
x x y y
INTERPRETACIÓN GEOMÉTRICA
La derivada parcial f x en el punto ( x0 , y0 ) representa la pendiente de la recta

tangente a la curva en x0 , y 0 , f ( x0 , y 0 )  . Es decir, es como si cortáramos la

superficie z = f (x, y) (en negro)


con el plano y  y 0 (en verde) y la

pendiente de la recta tangente (en


azul) de la curva intersección (en
rojo) es la derivada parcial de f
respecto de x en ( x0 , y0 ) . La

interpretación geométrica de la f y

es análoga, intercambiando el
papel de x e y.(ver gráfico).
Ejemplos
1. Determine f y (1, 2) y f x (1, 2) si f ( x, y)  x 3 y  5 y 4

Para calcular f x ( x, y) , debemos considerar y como constante y derivamos con

respecto a x, obteniendo: f x ( x, y)  3x 2 y  0  3x 2 y , y f x (1, 2)  3(1) 2 (2)  6

14
Borrador del Módulo de Matemática II

Análogamente, para hallar f y ( x, y) , debemos considerar x como constante y

derivamos con respecto a y, obteniendo: f y ( x, y)  x 3  20 y 3 , y

f y (1, 2)  (1) 3  20(2) 3  161 .

z z
2. Si z  x 3 cos( x 2  xy ) determine y
x y

z
x
 x3

x
  
cos( x 2  xy )  cos( x 2  xy )
 3
x
(x ) 
z
x
  

 x 3  sen( x 2  xy ). ( x 2  xy )  cos( x 2  xy ) .3x 2
x

 
z
x
  
 x 3  sen( x 2  xy ).(2 x  y )  cos( x 2  xy ) .3x 2 
z
  x 3 sen( x 2  xy ).(2 x  y )  3x 2 cos( x 2  xy )
x
z
y
 x3

y
   
cos( x 2  xy )  x 3  sen( x 2  xy ).(0  x)   x 4 sen( x 2  xy )

3. Un fabricante de un juguete popular a determinado que su función de

producción es P  LK donde L es el número de horas de trabajo por semana y

K el capital en cientos de dólares por semana requerido para la producción

semanal de P gruesas del juguete (una gruesa tiene 144 unidades). Determinar

las funciones de productividad marginal y evaluar cuando L=400 y K=16.

Interprete los resultados

Solución
P K P L
 y 
L 2 KL K 2 KL
Al evaluar estas derivadas cuando L=400 y K=16 tenemos
P 16 1
  y
L L  400
K 16
2 16 * 400 10

P 400 5
 
K L  400
K 16
2 16 * 400 2

15
Borrador del Módulo de Matemática II

El primer resultado significa que si se incrementa L a 401 y mantener fijo K en 16,


1
la producción aumentará en aproximadamente de gruesa. El segundo nos
10
dice que si K se incrementara a 17 y se mantiene fijo L en 400, la producción
5
aumentará en alrededor de gruesas.
2
NOTA: Las derivadas parciales obedecen las mismas reglas de derivación
habituales que las funciones de una variable, considerando la variable respecto a
la que queremos derivar como la única variable de la función, y las demás
variables como si fueran constantes
DEFINICIÓN (Derivadas parciales para n variables): Sea f : R n  R una
función en las variables ( x1 , x 2 , x3 ,...x n ) . Diremos que la derivada parcial de f con

respecto de x k en el punto ( x1 , x 2 , x3 ,...x n ) es la función:

f f ( x1 , x 2 ,...x k  h,...x n )  f ( x1 , x 2 ,...x n )


f xk ( x1 , x 2 ,...x n )   lim
x k h  0 h
siempre que exista el límite.

1.7. DERIVADAS DE ORDEN SUPERIOR


Al igual que en las derivadas ordinarias, es posible hallar las derivadas parciales
de una función de varias variables de segundo, tercer orden y superiores,
siempre que las derivadas existan.
Por ejemplo la función z  f ( x, y) tiene cuatro derivadas parciales de segundo
orden:
   f  2 f   f  2 f
   fxx     fyy
x  x  x 2 y  y  y
2

  f  2 f    f  2 f
    fyx    fx y
x  y  xy y  x  yx

NOTA: Las derivadas parciales f y x y fxy se les llama derivadas parciales

cruzadas de segundo orden de f, en el ejemplo anterior las derivadas parciales


cruzadas son iguales, esto no siempre se cumple.
Si las derivadas parciales f y x y f x y son continuas se cumple que f y x  f x y .

Ejemplos

16
Borrador del Módulo de Matemática II

1. Si f ( x, y)  x 5  xe y  xy 3  cos xy hallar las derivadas de segundo orden.

 f  f
 f x  5 x 4  e y  y 3  y sen xy  f y  xe y  3xy 2  x sen xy
x y

   f  2 f
   f x x  20 x 3  y 2 cos xy
x  x  x 2

  f  2 f
    f y y  xe y  6 xy  x 2 cos xy
y  y  y 2

  f  2 f
    f y x  e y  3 y 2  xy cos xy
x  y  xy

   f  2 f
   f x y  e y  3 y 2  xy cos xy
y  x  yx

2. Si f x, y   x3  3x 2 y  e5 x hallar las derivadas de segundo orden.

 f  f
 f x  3x 2  6 yx  5e 5 x  f y  3x 2
x y

   f  2 f
   f x x  6 x  6 y  25e 5 x
x  x  x 2

  f  2 f
    fy y  0
y  y  y
2

  f  2 f
    f y x  6x
x  y  xy

   f  2 f
   f x y  6x
y  x  yx

3. Si  
f x, y   ln x 2  y 2 hallar las derivadas de segundo orden.

 f 2x  f 2y
 fx  2  fy  2
x x  y2 y x  y2

   f  2 f
   f 
 
2 x 2  y 2  2 x2 x  2 x 2  2 y 2  4 x 2 2 y 2  x 2
 
 
x  x  x 2
xx
x2  y2
2
 
x2  y2
2

x2  y2
2
  
  f

 2 f
   f 
 
2 x 2  y 2  2 y 2 y  2 x 2  2 y 2  4 y 2
 
2 x2  y2  
y  y  y      
2 y y 2 2 2
x2  y2 x2  y2 x2  y2

  f  2 f  2 y2 x  4 yx
    fyx  
x  y  xy 
x2  y2
2

x2  y2  2

17
Borrador del Módulo de Matemática II

   f  2 f  2 x2 y  4 xy
   fx y  
y  x  yx x y
2 2 2

x  y2
2
  
2

DEFINICIÓN.- La función f : R2 
 R se llama función armónica se verifica la
2 f 2 f
ecuación de Laplace, es decir si:  0
x 2 y 2
EJEMPLO

1. Demostrar que la función f x, y   x 3 y  xy 3 es armónica.

 f  f
 f x  3 yx 2  y ,  f y  x 3  3xy 2
x y

   f  2 f
   f x x  6 yx
x  x  x 2

  f

 2 f
   f y y  6xy Por tanto la f x, y  es una función armónica.
y  y  y
2

1.8. DERIVACION DE LA FUNCION COMPUESTA

TEOREMA: Th “Regla de la Cadena”. Sea  R dada por z  f x, y 


f : R2 

donde f es una función derivable de x e y . Si x  g t  y y  ht  siendo g y h


funciones derivables en t , entonces z es una función derivable de t y
z z dx z dy
  .
t x dt y dt
EJEMPLO
z z  x 2  y 2 ; con y  et .
Hallar si x  e t ,
t
z z dx z dy
 
t x dt y dt
z z z
 2 x  t  2 y ( t )   2 x t  2 y t   2 t  t  2 t  t  4 2t
t t t

TEOREMA: Sea f : R2 
 R / z  f ( x, y) , donde f es una función diferenciable

de x e y . Si x  g (s, t) y y  h s, t  , donde las derivadas parciales x x y


, , ,
s t s

18
Borrador del Módulo de Matemática II

y z z z z dx z dy
existan todas, entonces , existen y están dadas por:  
t  s t  s x ds y ds
z z dx z dy
 
 t x dt y dt
EJEMPLOS

1. Hallar la z y z si
s t
 
z  x 2  y 2 con x
s 1
t
;y 
t 1
s
.

z z dx 1 dx s 1 dy t 1 dy 1
 2x ,  2y ,  ,  2 ,  2 , 
x y ds t dt t ds s dt s
z z dx z dy z z dx z dy
   
s x ds y ds t x dt y dt
z  1  2 yt  1 z 2 xs  1 2 y
 2 x    
s t  s2 s t2 s

z 2s  1 2t  1 z 2s  1 2t  1


2 2
   
s t2 s2 s t2 s2
x
2. Demostrar que z  arctg   , con x  u  v y y  u  v .
 y

z 1 1 y z 1  x  x
   2 ,     2    2
x x
2
y y  x2 y 2
x  y  y  x2
1    1   
 y  y
dx dx dy dy
1 ,  1, 1 ,  1
du dv du dv
z z dx z dy z z dx z dy
   
u x du y du v x dv y dv
z z
 2 2 1  2 2  1
y x
 2
y
1  2 x 2 1
u y  x 2
y x v y  x y x
z yx z yx
 2 2  2 2
u y  x v y  x
z z yx yx 2y 2u  v  2u  v 
  2  2  2  
u v y  x 2
y x 2
y x 2
u  v   u  v u  v 2  u  v2
2 2

z z 2u  v  uv
   2 2
u v 2(u  v ) u  v
2 2

19
Borrador del Módulo de Matemática II

GRADIENTE DE UNA FUNCIÓN

Si f : A  Rn 
 R es definida en un conjunto abierto A  R donde
  
D1 f  x , D2 f  x ,......Dn f  x  , existen entonces el vector gradiente de f se denota
     
       
por f  x   grad  f  , es definido por: f  x    D1 f  x , D2 f  x ......Dn f  x 
          

 f f f 
Si f : A  R 3  R , el gradiente de f es : f   , ,  .
 x y z 
PROPIEDADES DEL GRADIENTE:

Sea f , g : f : A  Rn  R diferenciables en A , entonces se tiene:


   
1.   f  g  
x  f  
x  g  x
     

     
3.  . f  x     f  x  
     

        
4.  f  g  x   f  x g  x   g  x f  x 
         

   


f  x g  x   g  x f  x 
 f            , 
5.   x   g x   0
 g     
2
 
 g x  
  

NOTA: Si f es una función diferenciables de x e y , la derivada direccional de


 
f en la dirección del vector unitario u es: D f x, y   f x, y   u .
u

NOTA: Entre todas las direcciones a lo largo de las cuales la función f crece la
dirección del gradiente cambia el signo, este señala la dirección de máxima
disminución.
EJEMPLOS

1. Hallar la derivada direccional de la f x, y   x  1y 2 e 2 xy en A0,1 la dirección

hacia B 1,3 .

20
Borrador del Módulo de Matemática II

f x, y   x 1y 2e2 xy


f
x
 
 x  1 y 2  e 2 xy  y 2e 2 xy  2 y 2 x  1e 2 xy

f
y
 
 y 2e 2 xy  x  1 2 y  e 2 xy  2 xy 2  e2 xy  2 yx  1e 2 xy  2 xy 2 x  1e 2 xy

 f f 

f x, y    ,   y 2e 2 xy  2 y 2 x  1e 2 xy ,2 yx  1e 2 xy  2 xy 2 x  1e 2 xy 
 x y 
f 0,1   1,2
    

u
AB

B A

 1,3  0,1   i  2 j   1  2   1 2 
i j  
  
, 
AB B A 5 5 5 5  5 5

   1 2 
D u f 0,1  f 0,1  u   1,2   
1 4 3
,    
 5 5 5 5 5

1.9. FUNCIONES VECTORIALES DE VARIAS VARIABLES


DEFINICIÓN.- La función f  A  R  B  R m se llama función vectorial de
n

variable vectorial.
EJEMPLOS

1. f ( x, y)  ( x  5, y  3) f : R2  R2

2. El gradiente de una f : R n  R es una función vectorial definida  f : R n  R n

f ( x, y, z)  x 2 y  yz f : R3  R
 f  (2 xy, x 2  z, y)
  
TEOREMA: f ( x, y, z)  f 1( x, y, z)i  f 2( x, y, z) j  f 3( x, y, z)k , entonces su
Si
 
f f1  f 2  f 3  f f1  f 2  f3 
derivada parcial se define como:  i j k,  i j k
x x x x y y y y

f f1  f 2  f3 
 i j k.
z z z z
EJEMPLO
  
1. Si f ( x, y )  e xy
i  ( x  y ) j  xsen ( y ) k

21
Borrador del Módulo de Matemática II

  
F x  ye xy i  j  sen( y)k
   
Fy  xe xy i  j  x cos( y)k
  
i j k
1 sen( y )  ye xy sen( y )  ye xy 1 
Fx  F y  ye xy 1 sen( y )  i  xy j k
 1 cos( y ) xe x cos( y ) xe xy 1
xe xy  1 x cos( y )
  
Fx  Fy  ( x cos y  seny)i  ( xye xy cos y  xe xy seny) j  ( ye xy  xe xy )k

DEFINICIÓN: Si f : A  R n  R m , es una función vectorial diferenciable en un
  
X  A con Fi diferenciables en X , a la función matricial de orden mxn definido
por:
  
 D1 f 1 ( x ) D2 f 1 ( x ) .......Dn f 1 ( x ) 
      
J ( F ( x ))   D1 f 2 ( x ) D2 f 2 ( x ) ......Dn f 2 ( x ) 
 D f ( x ) D f ( x ) .......D f ( x ) 
 1 m 2 m n n 
 
Es la derivada F en X , y se lo conoce como la matriz Jacobiana.
EJEMPLOS
Hallar la matriz Jacobiana de las siguientes funciones:

1. F ( x, y, z)  ( xy, y 2 , x 2 z)

 y x 0
   
J ( F ( x ))   0 2 y 0  Matriz Jacobiana
 2 xz 0 x 2 
 
y x 0

J ( F ( x, y, z ))  0 2y 0  2 y 2 x2 Jacobiano o determinante de la matriz
2
2 xz 0 x

Si F ( x, y)  ( x , y , e , xe , ye ) , hallar J en el punto (1,2).
y x xy y x
2.

F : R 2  R5
 yx y 1 x y ln x   2 0
 x   
 y ln y xy x 1   2 ln 2 1 
J ( F ( x, y, z ))   ye xy 
xe xy   J ( F (1.2))   2e
2
e2 
  2 
 ey xe y   e e2 
   2e
 ye
x
ex   e 

22
Borrador del Módulo de Matemática II

 
3. F ( r ,  )  ( r cos  , rsen ) , Halle el J ( F (r , )) Jacobiano

 cos   rsen 
J ( F (r ,  ))   J ( F (r , ))  r cos 2   rsen2
sen r cos 
 
J ( F (r , ))  r (cos 2   sen 2 )  J ( F (r ,  ))  r

NOTA: El valor J f , tiene una aplicación importante en las transformaciones

(cambio de variable), integrales dobles y triples.



DEFINICIÓN: Son f : A  R n  R m una función vectorial, si la matriz Jacobiana
 
J (F ) es continua sobre un conjunto abierto A  R n , entonces es f diferencia
sobre A.
DIVERGENCIA DE UNA FUNCION VECTORIAL

Si una función vectorial es f : ( f1 , f 2 , f 3 ) , donde f1 , f 2 , f 3 son funciones
escalares, entonces el producto escalar de la función vectorial f y el operador
  
 i j k se denomina divergencia de la función vectorial y se denota
x y z
 
por: f =div ( f ), y está definida por:
       
f  ( , , ) f 1, f 2, f 3  f  f1 f2 f3
x y z x y z

f f f
NOTA: f (u, w)  u  v  w
u v w
TEOREMA:
 
1. Si fy g son dos funciones vectoriales entonces la divergencia es

.( f  g )   f   g

 2  2  2
2. Si  es una función vectorial en la divergencia Div (grad  )=  
x 2 y 2 z 2

DEFINICIÓN: Una función escalar  se dice armónica si es continua, tiene


segundas derivadas continuas y satisface a la ecuación de Laplace:
 2  0

23
Borrador del Módulo de Matemática II

EJEMPLO

1. Analice si la función:  ( x, y, z )  x 2  y 2  z 2 , es armónica:

 2  2  2
Div (grad  )=  
x 2 y 2 z 2

 x  y
 x  ,  y  ,
x x2  y2  z 2 y x2  y2  z2

 z
 z 
z x2  y2  z 2

      2 y2  z2
   x x 
x  x  x 2 ( x 2  y 2  z 2 )3

      2 x2  z 2
   y y 
y  y  y 2 ( x 2  y 2  z 2 )3

   z   2 y2  z2
   z z 
y  y  z 2 ( x 2  y 2  z 2 )3

2
 2   No es función armónica
x2  y 2 z 2

ROTACIONAL DE UNA FUNCION VECTORIAL



Sea f  ( f1 , f 2 , f 3 ) con funciones escalares con primeras derivadas continuas

entonces el producto vectorial con está definido por:

i j k
  
 f 
x y z
f1 f2 f3
 
NOTA: El   f no necesariamente es perpendicular a f .
PROPIEDADES
1. f y g , entonces el x( f  g )  x f  x g

2. Si  es una función escalar con segundas derivadas continuas entonces


x( )  0
3. Sea f con segundas derivadas continuas entonces .(x f )  0

24
Borrador del Módulo de Matemática II

4. Sea f y g , entonces (x f ) g  (x g ) f

EJERCICIOS DE APLICACIÓN

u  3xe s  y
1 
1. Hallar el gradiente de la función f (u, v, w)  u v 
2
, si: w  x 2  y  1
w w  3 x 2  z  1

f f f
f  u  v  w
u v w
1

f f f 2 w 1
 2uv ,  u2 ,  
u v w w 3

2w 2

u (3e x  3xe x , 1, 0) , v (2 x,1,0) , w (6 x,0,1)

2. Si f ( x, y, z)  ( x cos z, yknx, z 2 e x ) , hallar el Jacobiano, Divergencia y Rotacional

cos z 0  xsenz
y
J ( F ( x, y, z ))  ln x 0  cos z (ln x)(2 ze x )  ( xsenz ) ln x( z 2 e x )
x
 z 2e x 0  2 ze x

J f x, y, z   2 ze x ln x cos z  z 2 xe x ln xsenz  ze x ln x(2 cos z  zxsenz) Jacobiano

 f f f
. f  1  2  3
x y z

. f  cosz   ln x   2 ze x Divergencia

  
i j k
      y
xf   (0  0)i  ( z 2 e x  xsenz ) j  (  0)k
x y z x
x cos z y ln x  z e
2 x

   y 
xf  0i  ( z 2 e x  xsenz ) j  k Rotacional
x

25
Borrador del Módulo de Matemática II

INTEGRALES MULTIPLES

2.1. INTEGRALES DOBLES


DEFINICIÓN: Sea f una función de dos variables definida en la región plana
R  a, b c, d   x, y  / a  x  b  c  y  d 

 
m n
Al lim  f xi , y j xi y j se le denomina la Integral Doble de f en R y se la
n
m j 1 i 1

  f x, y dxdy
d b
denota de la siguiente manera: .
c a

Además, si existe este límite decimos que f es integrable en R .


TEOREMA DE INTEGRABILIDAD: Sea f una función de dos variables definidas
en la región plana R  a, b c, d   x, y  / a  x  b  c  y  d .

Si f está acotada en R y si f es continua en R a excepción de un numero

finito de curvas suaves, entonces f es integrable en R .


TEOREMA FUBINI: Sea f una función de dos variables definidas en la región

plana R  a, b c, d   x, y  / a  x  b  c  y  d . Si f es continua en R,

 f x, y dA    f x, y dxdy   f x, y dydx


d b b
entonces:
c a a
R

EJEMPLOS:
5
5 4
5 2 
4 4 2 2 4
x dx   x 3   455
x y 21 2 21
1.   x ydxdy     x ydy dx   
4
2
dx 
2 1 1  2  1
2 2 2 1 6  1 
 2

b 2 2
a x  ba a2  x2 b 2 2
a x

 
a a
 
a a a
y2 a 1 b2 2
2.  0 xydydx  0  0  0 2
xydy dx  x dx   x 2 a  x 2 dx
20 a
0   0

b 2 2
a x
a a a a2

  xydydx  1 b 2  a 2 x  x 3dx  1 b 2 a 2 x  x   b 2  a  a 


2 a 2 2 4 2 4 4

0 0 2a 0 2a  2 4  0 2a  2 4 0

b 2 2
a x
a a

  xydydx  18 a b
0 0
2

26
Borrador del Módulo de Matemática II

EJERCICIOS DE APLICACIÓN
1 1

 y x
2
1. dxdy
0 0


y  x , y  x  0
2 2

yx  2
2


 x  y, y  x  0
2


y  x , y  x
2 2

yx  2 2


 x  y, y  x
2


R1  x, y   R 2 / y  x 2 

R2  x, y   R 2 / y  x 2 

 y  x  y  x dxdy   y  x 2 dxdy
2
2
dxdy 
R R1 R2

1 x2
   
1 1

 y  x
2
dxdy     x  y dydx    y  x 2 dydx
2

R 0 0 0 x2

x2 x2 1


1 1
y  x dxdy  x y
2 y2 y2
 dx  
1
2
  x2y dx
2 2 x2
R 0 0 0 0 x2

  
1
y  x dxdy   x  x  1  x  x 2 1  x 2 =  1 x  1 x 3  1 x 5   11
4 4
2 4
2 5  0 30
R 0
2 2 2  3

2. Calcular la integral doble  2 x  y dxdy , sobre la región R por arriba d


R

y  x  1 y debajo de y  4  x .

 x  1, x  1
y  x 1  
1  x, x  1
4  x, x  0
y  4 x  
4  x, x  0

27
Borrador del Módulo de Matemática II

 y  x 1 y  1 x
 
y  4  x y  4  x
0  2x  5 0  3  2 x
5 3
x x
2 2
5
0 4 x 1 4 x 2 4 x

 2 x  y dxdy    2 x  y dydx    2 x  y dydx    2 x  y dydx


R 3 1 x 0 1 x 1 x 1

2

5 4 x
4 x 2 4 x 4 x 2 4 x 4 x

 2 x  y dxdy    2 xy
0 1
y y 2
y2
 dy   2 xy  dy   2 xy  dy
R 3 2 1 x 0 1 x
2 1 x 1 x 1
2 x 1

2 1 x

 2
 dx   6x  12 4  x  
0 1

R 2 x  y dxdy  3 4 x  6  2 4  x   (1  x)


1
 (1  x) 2 dx
2 2 2


0 
2


 dx
2
  10 x  4  x   ( x  1) 2
1 2

… 1 2
5

 4 x 2  6 x  1 15  10 xdx  6 x  1 15  6 xdx  10 x  4 x 2  1 15  6 xdx


0 2 1

3  2 

0  2 

1  2 


2

 4 x 2  x  15 dx  9 x  15 dx  13x  4 x 2  15 dx


0 1 2

 3 
 2  0 
2 1  2

3

5
0 1
  4 x 3  1 x 2  15 x 9 15   4 13 15 
  x 2  x    x 3  x 2  x  
157 2

3 
 2 2  3  2 2 0  3 2 2 1 12
2

2.2. CALCULO DE AREAS Y VOLUMENES CON INTEGRALES DOBLES


EJEMPLOS

 xy  2 x dA ,
2
1. Calcular la integral doble
R

siendo R : y  x , y   x, x  0, x  4

0 x4
x y x

28
Borrador del Módulo de Matemática II

 xy  2 x dA     
x 
4 x 4
2
xy  2 x dydx    y 2  2 x 2 y  dx
2

0  x
R 0 x
2

 xy  2 x dA    2 xx  x   2 x  


1
4

4
1 1
5

2 2 2
x  x dx    x 2  x 3  2 x 2  2 x 3 dx
R 0   0
2 2 
4
 1 3 1 4 4 72 1 4 
R xy  2 x dA   6 x  8 x  7 x  2 x   21
2 3776
0

2. Calcular la Integral  x  2 y dxdy ,


D
donde D es la región limitada por las

 x 1 
rectas:  y  , y  3, y  1, x  7  .
 2 

Barrido Horizontal

1 y  3

2 y 1  x  7

 x  2 y dxdy    x  2 y dxdy
3 7

1 2 y 1
R

 x2 7
  49 2 y  12  
 x  2 y dxdy      
3
1  2 2 
3
  2 yx 2 y 1 dy
7
R x  2 y dxdy       2 y 7  2 y  1 dy
R
1
 2 2 y 1   

 49 2 y  12   3  49  4 y  4 y  1  8 y  32 y 
 x  2 y dxdy  
2 2
3
    4 y 2  16 y dy    dy

 2 2    2 
1 1
R

 x  2 y dxdy  2  48  36 y  12 y dy  2 48 y  18 y 2  4 y 3 


1 3
2
1 3 3 3

1 1 1 1
R

 x  2 y dxdy  2 144  48  162  18  108  4  68


1
R

3. Calcular el Área de la región comprendida

por R : y  x2 , y 2  x , por integración doble.

Barrido Horizontal

x2  y  x 0  x 1

29
Borrador del Módulo de Matemática II

AR    dxdy 
1 x

R

0 x 2
dydx

 3 1
1

 2x 2 x3   2 1  1 2
1

0 x2
x 1
dydx   y x 2 dy  
x 1
 
x  x dy  
2
     u
3 0  3 3 3
 3
0 0

 0 

4. Hallar el área de la región R, limitada por las curvas y  x 2  x, y  senx .

 b  b   1 1 
Vertice    , f       , 
 2 a  2a    2 4 

b 1 1 1 1
  , y  
2a 2 4 2 4

Barrido Horizontal

x 2  x  y  senx , 0  x  1

1 senx
  senx  x 
1 senx
AR    dxdy  dydx   y x dy 
1
2
x
2
 x dy
0 x x2
0 0
R

 1 1
x3
1
x2   1
1
1 1  2 1
AR    dxdy   cos x        1  1       
  3 0 2 0   3 2  6
R 0 

  12 2
AR    dxdy  u
R
6

5. Hallar el Volumen del cuerpo limitado por el paraboloide z  x 2  y 2 , los planos

coordenados y el plano x  y  1

30
Borrador del Módulo de Matemática II

Barrido Horizontal

0  x 1 0  y  1 x

V R    f x, y dxdy    x 
1 1 x
2
 y 2 dydx
0 0
R

 y3 
1 x
2 1 x
dx V R    f x, y dxdy 
1
  x y 
0
 0 3 0 
 R

1
   x 2 1  x  
1  x 3  dx

 3 
0

  4 x 3  6 x 2  3 x  1
V R    f x, y dxdy 
1
0 
1 1
 
 dx  0  4 x  6 x  3x  1 dx
3 2

R
3  3

  1 3 1
V R   1   x 4 1  2 x 3 1  3 x 2     1  2    u 3
1

3  0 0 2 0 3 2 6

EJERCICIOS DE APLICACIÓN
1. Por medio de la integral doble hallar la

región de la curva y 2  4  x , y 2  4  4x

2 4. y 2

 x
4 y 2
dxdy 
2
A( R)   dxdy 
R  
2
4 y 2
4
2 4 y 2
dy
4

1 2 1

4  2
(12  3 y 2 )dy  (48  16)
4
A(R)  8u 3

2. Calcular el volumen limitado por las superficies z  x2  y 2 , y  x 2 , y  1, z  0

31
Borrador del Módulo de Matemática II

Barrido Horizontal
1  x  1 , x2  y  1

V ( R)   f ( x, y )dxdy  
1 1

R

1 x 2
( x 2  y 2 )dydx

1 1 x6
  ( x 2  x 4   )dx
1 3 3
1 1 1 1 1 1 1 1 140  42  10 88 3
V (R)  (  )  (  )  (  )  (  )  u
3 3 5 5 3 3 21 21 105 105
3. Hallar el volumen de la región limitada por el paraboloide hiperbólico

z  x 2  y 2 y los planos z  0, x  3 .

z  0 x   y
 x2  y 2  x  y  
z  x  y  x y
2 2

Barrido Horizontal
0  x  3, x yx

V ( R)   dxdy 
3 x

R

0 x
( x 2  y 2 )dydx

x 3
y3 4 3 3 4 x4
dx   x dx  
3

x
V (R)  x y 
2
 27u 3
0 x 3 3 0 3 4 0
x

4. Hallar el volumen del solido limitado z  4  x2  y2 , z 2.

32
Borrador del Módulo de Matemática II

Barrido Horizontal

0 x 2, 0  y  2  x2
 V (R) 
3
2 (2  x 2 ) 2
4 2 2  x  x 2 2
2 x  2
dx
0 3
Cambio Trigonométrico:

x  2sen  dx  2 cos d  0   
2
Remplazar en 
2
x x  8 2 3
V (R)  8( 2  x 2  arcsen( ) )  4 2 2sen 2 2 cos 2 d   2 2 2 cos 4 d
2 2 0 0 3 0

1  cos 2 1  cos 2 32  2 1  cos 2


 2
V (R)  4  8 2
( )( )d   ( ) d
0 2 2 3 0 2
 1  cos 4 8  1  cos 4
V (R)  4  2 2 (1  )d   2 (1  2 cos 2  )d
0 2 3 0 2
  2
  2
8     8   3 3
V (R)  4  2( 0
2
 )  ( 0
2
 )  4  2(  )  (  )  u
2 0 3 2 0 2 4 3 2 4 2

2.3. INTEGRALES DOBLES EN COORDENADAS POLARES

 f ( x, y)dA   f (r, ) rdrd 


R R

1. Hallar el volumen del solido limitado z  4  x2  y2 , z  2

33
Borrador del Módulo de Matemática II

 z2
  x2  y2  2
z  4  x  y
2 2

x  r cos 
y  rsen

x2  y2  r 2  r  2
Región de Integración
0r  2
0    2
2
2 2 r4 2
 d   d  2u 3
2 2
V   (2  r )rdrd 
2
r 2

0 0 0 0 4 0
0

2. Encuentre el volumen de la región limitada por las superficies

x 2  y 2  z 2  4, x 2  ( y  1) 2

x  r cos 
y  rsen
x 2  y 2  2 y  r  2sen

0  r  2sen
0  
3
 2 sen 2  3
V  4  r 2 rdrd    (4) 2 d
2
( 4  r 2
)
0 0 3 0
16  16  16
V
3 0
(cos   sen 2 cos   1)d   ( 0 )  u 3
3 3

34
Borrador del Módulo de Matemática II

3. Evaluar 
dxdy
donde x 2  y 2  2x  0
R 4  x2  y2

x  r cos 
y  rsen
x 2  y 2  2 x  r  2 cos 

0  r  2 cos 
0  
 2 cos rdrd  2 cos


1
 1 (4  r 2 ) 2
d
0 0
4  r2 0 0

0


2 cos

0
rdrd
4r 2

 2 ( sen  2)d   2  cos 
0
 
0


0
  2  4
4. Calcular  ydxdy , D : r  1 cos  , región encerrada por el cardiode.
D

x  r cos  y  rsen
0   0  r  1  cos 
1 cos
 1 cos  r3
 ydxdy   
D
0 0
rsenrdrd   0 3
sen d
0

1 
 ydxdy 
D
3 0
(1  cos  ) 3 sen d

u  1 cos   du  send

1  3 1 (1  cos  ) 4 1 4
D 3 0
ydxdy  u du    (2) 4 
3 4 0
12 3

 
( x2  y 2 )
5. Calcular dxdy , donde D es la región del primer cuadrante
D

x 2  y 2  a 2 y los ejes coordenados.

35
Borrador del Módulo de Matemática II

x  r cos  y  rsen

0r a 0  
2

a
  e r rdrd Sustitución básica u  r 2  du  2rdr
2
2
0 0

  a  a
e ( )dud   1  2 e r d   e  1  2 d   e  1 
2 2
a 1
 
2 u a 2

0 0 2 2 0 0 2 0 4

6. Hallar el volumen de la región en el espacio limitado superiormente por el cono

z x 2  y 2 dentro del cilindro x 2  y 2  1 y sobre el eje x.

x2  y2  1  r  1

0  r 1  0  
1
  r3 1  1  
V (R)   f ( x, y)dxdy    r rdrd   d   d   0  u 3
1
2

D
0 0 0 3 0 3 0 3 3

7. Hallar el área de la región plano D ubicado en el interior del circulo r  3 cos  en

el exterior del cardiode r  1 cos 

36
Borrador del Módulo de Matemática II

3 9
x 2  y 2  3x  ( x  )  y 
2 2

2 4
 r  3 cos  
  3 cos   1  cos   2 cos   1   
r  1  cos  3

0   , 1  cos   r  3 cos 
3
  3 cos 
3COS
A(R)   dxdy  2 3
 r.drd   r 3 2
d   3 (8 cos 2   2 cos   1)d
0 1 COS 0 1 cos 0
R


  
sen2 3 4 
A(R)  4( 3
0
  2sen 3
0
  03 )   3  3   u 3
2 0 3 3

2.4. INTEGRAL DOBLE CAMBIO DE VARIABLE


La fórmula para la transformación de integrales dobles, utilizando un cambio de
variable se puede escribir así:  f ( x, y)dxdy   f (u, v) J (u, v) dudv
D S

x x
( x, y ) u v
J (u, v)  
(u, v) y y
u v
1. Si R es la región triangular del plano XY limitado por x  0, y  0, x  y  1 , y

x y

 e
x y
la integral dydx
R

u  x  y uv v u
  x  y
v  x  y 2 2

37
Borrador del Módulo de Matemática II

1 1
J (u, v)  2 4 1
1 1 2

2 2
x  0  u  v
y  0  v  u
0  v 1
v  u  v
x y u v
(e  e 1 ) 1
u u
1 1 v1 1 v
 e
x y
dxdy   e J (u, v) dudv    e
v
dudv   vev
dv  0 vdv
0 v 2 2 0 2
r S v

x y 1
(e  e 1 ) v 2 (e  e 1 )
 e
x y
dxdy  
r
2 2 0
4

y 2 cos xy
2. Calcular la integral doble 
D
x
dA , donde D es la región limitada D:

 y  x2

x y
2

 2
x  4 y

 y  4x
2

1 u  4
1 v  4
y2 2y

J ( x, y )  x x  1  4  3
2x x2
 2
y y
1
J (u, v)  
3

38
Borrador del Módulo de Matemática II

y 2 cos xy 1 1 4 4


4 1 4

D
x
dA  
1 1 u cos(u.v )
3
dvdu 
3 1
sen(u.v )
1
dv  1 sen(4u)  sen(u)du
4
y 2 cos xy 1 cos 4u 1 cos 16 cos 4
 dA  (  cos u 1 )  (
4
  cos 4  cos 1)
D
x 3 4 1 3 4 4

y 2 cos xy 1

D
x
dA  (5 cos 4  4 cos 1  cos 16)
12

 y  2x
(2 x  y )  y  12  4 x
2

3. Calcular  dxdy si D: 
D
1  4 xy  y  y  4x

 y  2x  2

2 1
J ( x, y)   2  4  2
4 1
1
J (u, v)  
2
(2 x  y ) 2 1 2 12 u 2
D 1  4 xy  y dxdy 
2 0 0 1  v
dvdu

(2 x  y ) 2 1 2 2 ln 13 2 2
D 1  4 xy  y 
2 0
 
2 0
12
dxdy u ln(1 v ) 0
du u du
2
ln 13 u 3 4 ln 13
( )
2 3 0 3

 cos(2 x  y) 
 2( x  y) 2 dA, siendo D la región
2
4. Evaluar la integral doble sobre
D

en el primer cuadrante acotado por 2 x 2  y 2  4 y

los ejes coordenados

 x
  r cos 
x 2
y  2
2  x  2r cos 
 1    
2 4  y  rsen  y  2rsen

2

39
Borrador del Módulo de Matemática II

r 2 cos 2   r 2 sen 2  1 r2 1  r 1

0  r 1 0  
2
x x
J (r , )  r   2 cos   2rsen
 2 2r
y y 2sen 2r cos 
r 
El argumento de la función integrante dejar en función de r y 
(2 x  y) 2  2( x  y) 2  4 x 2  4 xy  y 2  2 x 2  4 xy  2 y 2
(2 x  y) 2  2( x  y) 2  6 x 2  3 y 2

(2 x  y) 2  2( x  y) 2  3(4r 2 cos 2   4r 2 sen 2 )

(2 x  y) 2  2( x  y) 2  12r 2 (cos 2  sen 2 )

(2 x  y) 2  2( x  y) 2  12r 2

  cos3(2( 

1
 cos(2 x  y )dA  2r cos  ) 2  (2rsen ) 2 ) 2 2
2 2 2
0 0
R


1
 cos(2 x  y )dA  2 2  2  cos(12r )rdrd
2 2 2
0 0
R

Sustitución básica u  12r 2  du  24rdr



2 2 2 1 2 2 2
R cos(2 x  y )dA  24 0 0 cos udud  12 0 sen(12r ) 0d  12 sen(12)02 d
2 2 2 1

2 
 2sen(12)
 cos(2 x  y )dA  sen(12) 
2 2 2
0
R
12 24

x y
5. Calcular la integral doble  cos x  y dxdy, donde
R
R es la región limitado por

ñas rectas x  y  1, x  y  4, x  0, y  0 .

40
Borrador del Módulo de Matemática II

u  x  y uv uv
  x  y
v  x  y 2 2

Ux Uy 1 1
J ( x, y)    11  2
Vx y 1 1
1 1
J (u, v)  
J ( x, y ) 2
1 v  4  v  u  v
v
x y u 1 1 4 u
v cos v 2 dudv  2 1 v sen v
1
sen(1)  sen(1)1 vdv
4 v 4
R cos x  y dxdy   1
v
dv 
2
4
x y v2 15
R x  y
cos dxdy   sen(1)
2

2
sen(1)
1

6. Calcular la integral 
D
xy dxdy , donde D es un dominio limitado por la línea

x 2 y 2 4 xy
(  )  , situado en el primer cuadrante.
2 3 6

x  2u  y  3v , (u 2  v 2 ) 4  uv

41
Borrador del Módulo de Matemática II

1
0
1
J ( x, y )  2   J (u, v)  6
1 6
0
3
3 3

 xy dxdy   2u 3v 6dudv  6 4  uv dudv  6 4  (u 2  v 2 ) 2 dudv 


D D D D

Coordenadas polares:
u  r cos   v  rsen
 sen(2 ) 6
1
0   0r( )
2 2
Reemplazar en 
1
sen 2 6 3

1
3 3 sen 2 6 3  6 ( )
r 2 64
 (u  v ) dudv  6 
( )
  r 5 drd  6  d  sen2 d
4 2 2 2
6 4 2 2 4 2 2

D
0 0 0 6 0
12 0

3 3
3 
64 64 1
6  (u 2  v 2 ) 2 dudv 
4
( cos 2 ) 02   6
D
24 6.2 2 6

2.5. INTEGRALES TRIPLES


f db l m n
DEFINICIÓN:  f ( x, y, z )dxdydz  Lim f ( x i , y j , z k )xi y j z k
n k 1 j 1 i 1
e ca m
l 

Siendo f ( x, y, z)  1, la Integral Triple representa el volumen

EJERCICIOS DE APLICACIÓN
1. Calcular  (2 x  3 y  z)dxdydz , si
T
T es el prisma triangular limitado por los

planos z  0 , z  a , x  0 , y  0 , x  y  b ; (a  0)( y  b) .

42
Borrador del Módulo de Matemática II

0  x  b, 0 y bx
a
 (2 x  3 y  z)dxdydz 
T
 
R
0
(2 x  3 y  z )dzdxdy

 z2
a

  2 xz 0  3 yz 0  dxdy
a a

R  0
2
 

a2
 (2 x  3 y  z)dxdydz 
T
 (2ax  3ay 
R
2
)dxdy 

b b x 1  3y 2
b x
y
b x 
a 
b x b

0 0 2 T
(2 x  3 y  )dydx ( 2 x  3 y  z ) dxdydz  a 0  0
 2 xy 
2

2
dx

 0 0

b 3(b  x) 2 b  x  a b 2
 a  2 x(b  x) 
0
 2

2 
dx 
2 0
3b  2bx  b  x  3x 2 dx 

a 2 b  a  3 b2 
b
x2
 (2 x  3 y  z)dxdydz 
b 3 b
3b x  bx 2  bx 0   3x   3b  
b

2 0 0 2 0
 2  2
 0 T

ab 2 (6b  1)
4
2. Calcular  xyzdxdydz , donde
T
T es la región

limitada por: x  y 2 , x 2  y , z  0 , z  xy .

0  z  xy , 0  x  1 , 0  y  x
 xyz 2 xy

T xyzdxdydz  R 0 xyzdz dxdy  R  2
xy
 dxdy 
 0 
1 1 1 x
  x 3 y 3 dxdy    2 x 3 y 3 dydx
2 R 2 0 x

1  x3 x8  1  1 1  1
 
1 1
1 1 4 x
1 1
T xyzdxdydz   8 0 y dx  
4
x  x dx  8
   
x2 8 0 8  3 0 9 0  8  3 9  96
 

3. Encuentre el Volumen del solido limitado por

arriba por el paraboloide z  4  x 2  y 2 y por

abajo que el plano z  4  2 x

43
Borrador del Módulo de Matemática II

z  4  x 2  y 2
  ( x 2  2 x  1)  y 2  1  ( x  1) 2  y 2  1  C (0,1)
 z  4  2 x

   x 2  y 2  2 x dxdy 
 4 x  y dz  dxdy  z 4 x  y dxdy
2 2 2 2

V ( R)   dxdydz   
R 
4  2 x 
 
T R 4 2 x R

Coordenadas polares:
x  r cos 
y  rsen
x 2  y 2  2 x  r  2 cos 

0  r  2 cos 
0  
Remplazar en 
2 cos 2 cos
 2 cos  2r 3 cos  r4 4 
V (R)    (2r cos   r )rdrd    d   cos 4  d
2
0 0 0 3 0
4 0
3 0

 
4  1  cos 2 2 1  1  cos 4 1    sen4 
V (R)  
3 0
(
2
) d  
3 0
1  cos 2 
2
d  
3 
 0
 sen 2 0

20

8

0

1  
V (R)     
3 2 2

2.6. INTEGRALES TRIPLES EN COORDENADAS CILÍNDRICAS


Si f : S  R 3  R es una función continua sobre S, entonces la transformación

de la integral triple  f ( x, y, z )dxdydz en coordenadas cilíndricas está dado por:

 f (r , , z ) J (r , , z ) dzdrd
s
La relación entre coordenadas cilíndricas y cartesianas es:
 x  r cos 

 y  rsen
z  z

xr x xz cos   rsen 0
J (r ,  , z )  y r  sen r cos  0  r cos   rsen   r
2 2
y yz
zr z zz 0 0 1

44
Borrador del Módulo de Matemática II

NOTA: Suponiendo que S es un sólido en coordenadas cilíndricas y que f (r , , z )

representa la densidad en cada punto r , , z  , entonces la masa del cilindro S se


calcula mediante la Integral Triple y está dado por:
M   f (r , , z )dzdrd
s

Si f (r, , z)  1 r , , z   S , se obtiene el volumen del solido S, es decir:

V ( S )   rdzdrd
s

EJERCICIOS DE APLICACIÓN

 x 
 y 2 dxdydz , donde T
2
1. Calcular
T

limitado por las superficies

x2  y2
z , z2
2

 z  2
z  x  y
2 2
 x2  y 2  4
 2
r 2 (cos 2   sen 2 )  4  r  2

r2
0r 2, 0    2 , z2
2

 2 2 
 x 
 y 2 dxdydz  R r22 r rdzdrd 
2

2 2  3 r5 
  r z r drd   0  2r  2 drd
3 2
2
0
R 2

2 2
 16  8 2 16 3
 x  2 r4 r6 2
 y dxdydz    d    8  d   0 
2 2
u
T
0 2 0 12 0 0
 3 3 3

2. Calcular el volumen de la parte del cilindro x 2  y 2  2ax, comprendido entre el

paraboloide x 2  y 2  2az y el plano xy .

45
Borrador del Módulo de Matemática II

r2
0  r  2a cos  , 0     , 0  z 
2a
r2 r2 2 a cos
 2 a cos  2 a cos  2 a cos r3  r4
V (R)  
0 0 
0
2a rdzdrd  
0 0
z 2a
0
drd  
0 0 2a
drd  0 8a 0
d

  1  cos 2 
2
16  4 a3  1  cos 4
V (R) 
2a 0
   0  2 
     1  cos 2  d
4 3
a cos d 2 a d
2 0 2
 
a3     cos 4  a3    3a 3 3
V (R)  
 0  sen 2        4 u
 
0
2  20 8 0
 2 2

2.7. INTEGRALES TRIPLES EN COORDENADAS ESFERICAS


Si una S : R  R 3 tiene un eje de simetría la integral triple se puede calcular en
forma muy simple uniendo las coordenadas esféricas cuya relación entre las
coordenadas cartesianas es: x   c o ss e n , y  sensen , z   cos 
 : El Angulo de la proyección del vector   0,2 
 : Es el ángulo que forma el vector con el eje positivo   0, 

Si S  R 3  R es una función continua sobre S, entonces la transformación de la


integral triple  f ( x, y, z)dxdydz
T
en coordenadas esféricas está dado por:

 f (  , , ) J (  , , ) ddd    senddd


2

S S

cos sen .sensen p cos  cos 


J (  , ,  )  sensen  cos sen sen cos    2 sen
cos  0  senp

Si f (  , ,  )  1 se obtiene el volumen del solido S, es decir:

V ( S )    2 senddd
S

46
Borrador del Módulo de Matemática II

EJERCICIOS DE APLICACIÓN
1 4 y 2 4 x 2  y 2 dzdxdy
1. Evaluar 
0 0 
0 x  y2  z2
2
, usando

coordenadas esféricas


0 
 0 y2 2
 
 0  x  4 y
2
 0  
0  z  4  x 2  y 2 2
 0  2
   
4 y 2 4 x 2  y 2 dzdxdy 2  2 sen
   ddd   2  2 sen 0 dd 
1
 
2
 2 2
0 0 0 x  y2  z2
2 0 0 0  2 0 0

  
  2 cos  02 d  2 2 d  
2
0 0

2. Encontrar el volumen del solido acotado por la

esfera x 2  y 2  z 2  36 usando

coordenadas esféricas

0    6 , 0    2 , 0    

6
2  2  3
V (R)     
6
senddd     senddd  sen dd
2 2

R
0 0 0 0 0 3 0

2  2  2 2
V (R)  72  sendd   72 cos  0 d   72(1  1)  d  144 0
 288u 3
0 0 0 0

2.8. APLICACIÓN DE INTEGRALES TRIPLES EN COORDENADAS


CILINDRICAS Y ESFERICAS
4 3 9 x2
1. Evaluar la Integral   x 2  y 2 dydxdz .
0 0 0

0  y  9  x 2

D : 0  x  3
0  z  4


x  r cos  , y  rsen , z  z

47
Borrador del Módulo de Matemática II


0  r  3 , 0   , 0 z4
2
xr x xz cos   rsen 0
J (r ,  , z )  y r y y z  sen r cos  0  rsen   r cos   r
2 2

zr z zz 0 0 1

f r ,  , z  J r ,  dzdrd 
9 x 2 4 3
  
4 3
 x  y dydxdz  r.rdrddz
2 2 2
0 0 0 0 0 0
s
  3
9 x 2 r3 4  9
ddz  9  02 dz  
4

4 3 4
  r drddz   
4 3
 x  y dydxdz 
2 2 2
2 2 dz
0 0 0 0 0 0 0 0 3 0 0 2 0

4 3 9 x 2 9 4

0 0 0
x 2  y 2 dydxdz  z  18
2 0
2. Si D es la región limitada por los planos x  1, x  2 por los cilindros

y 2  z 2  4, y 2  z 2  9 . Calcular  e x y 2  z 2 dxdydz .


D

x  r cos  , y  rsen , z  z
Coordenadas Cilíndricas
2  r  3 , 0    2 , 1  x  2
xr x xz cos   rsen 0
J (r ,  , z )  y r y y z  sen r cos  0
zr z zz 0 0 1

 rsen2  r cos 2   r

f r ,  , z  J r ,  dzdrd 
2 2

3 2 3 2

s
 
0 2 1
e x y 2  z 2 dxdydz   0 2 1
e x r.rdxdrd

2 3 2 2 3 2 2 3
     e
x 2
e x y 2  z 2 dxdydz  e x r 2 dxdrd  r 2 drd
0 2 1 0 2 1 0 2 1

   r drd  e  r3
2 3 2 2 3 2
  y  z dxdydz  e  e e d
x 2 2 2 1 2 2 1
e
0 2 1 0 2 0 3 2
2
 
3 2
e x y 2  z 2 dxdydz  e  e
2 1
  2  8
 9  d 
19 2
e  e1   2
d  
19 2
e  e1 
2

 3 0
0 2 1 0 3 0 3

 e 2  e1 
2 3 2 38
 
0 2 1
e x y 2  z 2 dxdydz 
3

48
Borrador del Módulo de Matemática II

3
 x2 y 2 z 2  2
3. Calcular  1  2  2  2  dxdydz donde
s 
a b c 

S es la región encerrada por el elipsoide

x2 y2 z 2
   1.
a 2 b2 c2

Coordenadas Esfericas
x
 a   cos  .sen  a cos  .sen

y
D :   sen .sen  bsen .sen
b
z
 c   cos   c cos 

Coordenadas cilíndricas
0    1 , 0    2 , 0    

x x x a cos  .sen  asen .sen a cos  . cos 


J (  , ,  )  y  y y  bsen .sen b cos  .sen b cos  . cos   abca 2 sen
z z z c cos  0  csen
3
 x2 y 2 z 2  2
 f r ,  , z  J r ,  dzdrd   1  2  2  2  dxdydz
s 
s
a b c 
3
 x2 y 2 z 2  2
 
2  1 3

s  a b c 
 1  2
 2
 2
 dxdydz  0 0 0 1   2 2
abc 2 sen.ddd

2.9. INTEGRALES CURVILINEAS O DE LINEA


DEFINICION: Si existe un número L talque   0,   0 y
n

 f x ' , y ' , z 'S


i 1
i i i i  L   para toda partición con Si   , entonces existe la

integral curvilínea de f con respecto a la longitud de arco Si y lo

representaremos por:
n

 f x, y, z ds  lim  f x ', y ', z 'S


i i i i L
S i  0
C i 1

49
Borrador del Módulo de Matemática II

PROPIEDADES FUNDAMENTALES DE LA INTEGRACIÓN CURVILÍNEA



PROPIEDAD 1. Consideremos una curva regular  : a, b 
 R3 , definida por

 
 t    1 t ,  2 t ,  3 t  tal que  a, b  C  R 3 es la imagen de  . Si
 

 

f : C  R 3  R , una función continúa sobre C, entonces la integral curvilínea


b  

 f x, y, z dS   f  t   ' t  dt , en donde la norma de la derivada está definida
C
a
 
2 2 2

     
por:  ' t    '1 t    ' 2 t    '3 t  , esta integral recibe el nombre de
     
Integral Curvilínea de Primera Especie.

EJEMPLOS

 x n
1. Calcular la Integral Curvilínea 2
 y2 ds , donde C es la circunferencia
C

x  a cos t , y  asent .
 
 t   a cos t , asent    ' t    asent, a cos t 

0  t  2   : 0,2  
 R 2

 ' t    asent 2  a cos t 2  a 2 sen 2t  a 2 cos 2 t  a

 x   a sen t  a 
n 2 n
2
 y 2 ds  2 2 2
cos 2 t adt 
0
C

a 2 n1dt  a 2 n1 t 0  2 a 2 n1 


2 2
0

 x 
 y 2  z 2 dz , donde la curva es definida por
2
2. Calcular la Integral Curvilínea
C


C:  t   cos t , sent , t  , 0  t  2 .


0  t  2   : 0,2  
 R3
 
 t   cos t , sent , t    ' t    sent, cos t ,1

50
Borrador del Módulo de Matemática II


 ' t    sent 2  cos t 2  1  sen 2 t  cos 2 t  1  2

2
 
 x   sen t  cos  1  t dt  2 t 02  t3
3
2 2
2
 y  z dz 
2 2 2 2
t t 2
2dt  2 2

C
0 0
 0 

 8 3   6  8 3  2 2
C x  y  z dz  2 2  3   2  3   3 3  4 
2 2 2 2

ds
3. Calcular la integral curvilínea x
C
2
 y2  z2
, donde C es la primera espira de la

hélice circular: x  a cos t , y  asent , z  bt .



0  t  2   : 0,2  
 R 3
 
 t   a cos t , asent , bt    ' t    asent, a cos t , b

 ' t    asent 2  a cos t 2  b2  a 2 sen 2t  a 2 cos 2 t  b 2  a 2  b2

ds 2 dt 2 dt
x
C
2
 y2  z2
 
0 a cos t  a sen t  b t
2 2 2 2 2 2
a 2  b2  a 2  b2 
0 a  b 2t 2
2

2
ds 2 dt a2  b2 2 dt a 2  b2  bt 
C x 2  y 2  z 2  a  b 0    arctg  
2 2

 b 2t 2  a2 0  b 2t 2  a 2
 a 0
a 2 1  2  1  2 
 a   a 

ds a 2  b2 a  2b  a2  b2  2b 
C x 2  y 2  z 2  a 2
 arctg 
b  a 

ab
 arctg 
 a 

NOTA: Cuando se tiene una curva plana y  x , a  x  b y f x, y  es una


función continúa, entonces la integral curvilínea se calcula mediante la fórmula:

 f x, y, z dS   f x, x 1   x dx


b 2
a
C

4. Calcular la integral curvilínea y sen 3 x 1  cos 2 x dS , donde C es el arco de la


2

curva y  senx , desde el punto 0,0 hasta el punto   ,1 .


2 


0 x
2

51
Borrador del Módulo de Matemática II

f x, y, z dS   f x, x 1   x  dx


b 2
dS  1  cos 2 x
a

y sen x 1  cos x dS   sen 2 x.sen 3 x 1  cos 2 x 1  cos 2 x dx


2 3 2 2
0
C

 

y
2
sen x 1  cos x dS 
3 2
0
2 sen x. 1  cos x 1  cos x dx 
5 2 2
0
2 
sen 5 x. 1  cos 2 x dx 
C

y  1  cos x .1  cos xsenx.dx


2
2
sen x 1  cos x dS 
3 2 2 2 2
u  cos x 
0
C

du  senx
 

y sen x 1  cos x dS   1  u  .1  u du   1  u 


2 2
 u 4  u 6 .du
2 3 2 2 2 2 2
0 0
C

  3
  
  
  

u 2  u 2 u5 2 u7 2   cos 3
x 2 cos 5
x 2 cos 7
x 2

        cos x 02   
2 3 2
y sen x 1 cos x dS
 0 3 5 7 0  3 0 5 0 7 0
   
C 0 0

 1 1 1  64
y sen 3 x 1  cos 2 x dS     1     
2

C  3 5 7  105

PROPIEDAD 2: Consideremos una curva regular  : a, b 
 R3 definida por:

 
 t    1 t ,  2 t ,  3 t  tal que  a, b  C  R 3 es la imagen de 
 
Si
 

P, Q, R : C  R 3  R , una función continua sobre C, entonces la integral curvilínea


       
 Px, y, z dx  Qx, y, z dy  Rx, y, z dz   P t  't   Q t  't   Q t  't dt
b
1 2 3
a
C

Estas integrales recibe el nombre de Integral Curvilínea de Segunda Especie.

EJEMPLOS
1. Calcular la Integral Curvilínea  ydx  xdy , donde C es el primer cuadrante de
C

x  R cos t y  Rsent desde t1  0 hasta 


la circunferencia t2  .
2

 
 
0t   : 0,  
 R 2
2  2
 
 t   R cos t , Rsent   ' t    Rsent , R cos t 

52
Borrador del Módulo de Matemática II

 

 ydx  xdy   Rsent  Rsent   R cos t R cos t dt    R sen t  R 


2 2 2
2 2
cos 2 t dt
0 0
C


   
 ydx  xdy  R     sin 2t 
 sen t  cos t dt  R  cos 2t dt  R 
2
0
2 2 2 2 2 2 2
0 0
 2 
 
C 0

x 2 dy  y 2 dx
2. Calcular la Integral Curvilínea  5 5
,
C
x y
3 3

donde C es la cuarta parte del astroide x  R cos t


3

y  Rsen 3t desde el punto R, 0 hasta 0, R  .

 
 
0t   : 0,  
 R 2
2  2

 t   R cos 3 t , Rsen 3t   ' t    3R cos 2 tsent ,3Rsen 2 t cos t 


 

x 2 dy  y 2 dx
  2
 
 R cos t 3Rsen t cos t  R sen t  3R cos tsent
6 2 2 6 2
 dt
  2
 
   
5 5 0 5 5
C
x y
3 3
 R cos t  Rsen t
3 3 3 3

x 2 dy  y 2 dx   3 2  4 
 sen 2t cos 2 t sen5t  cos 5 t
 R cos 5tsen t  R 5sen t cos t  dt  3R 3  2 
7 2 3 7
 dt
 5 5
 
0
2
  0
 cos 5 t  sen5t   

C
x y
3 3  R cos t  R sen t
3 5 3 5


x 2 dy  y 2 dx 4 
 
4 
1  cos 4t 
 3R 3  2 sen 2t cos 2 t dt  3 R 3  2 sen 2 2t dt  R 3  2 
4
3
 5 5 0 4 0 4 0
 2
dt

C
x y
3 3


4 
x 2 dy  y 2 dx   3 R 3     3 R 3
4 4
3 3 2 3 3  2 sen4t
4

  R  1  cos 4t dt  R t 0 
2

8   8  2  16
5 5
8 0 4
C
x3  y3 
0

xy 2 dy
3. Calcular la Integral Curvilínea 
C
x2  y 2
, a lo

largo del círculo x 2  y 2  a 2 en sentido anti

horario.

 x  a cos  
 0    2  : 0,2  
 R 2
 y  asen

53
Borrador del Módulo de Matemática II

 
    a cos  , asen   '     asen , a cos  

xy 2 dy 2  a cos  asen 2 a cos   2  a 4 cos 2 sen 2 


 x2  y2 
C

0

 a cos    asen  
2 2 
d  0  2 2 2 2 
 a cos   a sen  
d

 a 4 cos 2 sen 2 
xy 2 dy
   sen 
2 2 a2 
 x2  y2   
2

   0     2 d
2 2 2 2
 2  d a cos sen d
 a cos   sen 
2 2
C
0
 4 0

2
xy 2 dy a2 2 1  cos 4  a2 2 a 2  2 sen4 
 x2  y2 4    d   1  cos 4 d   0  
C
0
 2  8 0 8  4 0 
xy 2 dy a2 a2
 x2  y2 8 2   
C 4

NOTA: Si Px, y  y Qx, y  son funciones continuas e y   x  , a  x  b es la


ecuación de una curva plana C, entonces la integral curvilínea se calcula

 Px, y dx  qx, y dy   Px, x  Qx, x ' xdx
b
mediante la fórmula: a
C

 y dx  x dx , donde C es el Segmento de la
2 2
4. Calcular la Integral Curvilínea
c

recta y  2 x  3 de A 1,1 hasta B2,7 .

 y  2x  3
c: 1  x  2
 y '  2dx

 y dx  x dx   2 x  3 dx  2 x dx   6 x 
2 2
 12 x  9 dx
2 2 2 2 2
1 1
c

 6 x3 2
12 x 2
2

c y dx  x dx   3   9 x 1   18  18  27  63
2 2 2

 1
2 1 

x 2 dx 2 ydy
5. Calcular la Integral Curvilínea 
c x2  y2

4x 2  y 2
,

de 0, 0 hasta 2, 2 .


x2
donde C es el arco de y 
2

x2
y dy  xdx 0 x2
2

54
Borrador del Módulo de Matemática II

 
 x2   
x 2 dx 2 ydy 2 x 2 dx
2 2 xdx   xdx xdx 
  
2
0  1 2
 2  2  
 
x y
2 2 4 x  y 2 0
 2  x2 
2
 x 2 2
  1 2
4 x
4 x 2      1  4 x
c
 x    4 
  2   2  

 2
 1 2 
2
x 2 dx 1 2 5
  4 1  x  2 ln  4  x    4  2 ln 5  2 ln 4  4  2 ln  
2 ydy
 x y
2 2
 2
4x  y 2
 4 0  4  0  4
c

 
t  a, b

PROPIEDAD 3: Si la curva C :  t   1 t ,  2 t ,  3 t  para una curva
 
seccionalmente regular entonces una partición a  t  t1  t2 .....  tn  b , existe para

a, b talque C, resulta ser la unión de las curvas regulares C : C1  C2  C3.....Cn ,


donde C1 : 1 t , t  a, t1 , 2 t , t  t1 , t2 ....... n t , t  tn1 , b y sea f : C  R3 
 R
una función definida en C, entonces se tiene:

 f x, y, z dS   f x, y, z dS   f x, y, z dS  .....   f x, y, z dS


C C1 C2 Cn

2.10. LONGUITUD DE ARCO

Sea f y f ' continúas en a, b, donde y  f x comprendida entre A  a, f a  o

B  b, f b donde: L   1  f ' x  dx .


b 2
a

TEOREMA: f : R 
 R n , tiene una derivada continua sobre el intervalo a, b y
la curva C descrita por f t  es rectificable entonces la longitud de aro de la curva

f ' t  dt
b
C es L  a

55
Borrador del Módulo de Matemática II

NOTA:Si f t    f1 t , f 2 t , f 3 t  con a  t  b , entonces L    f ' t    f ' t    f ' t 


b 2 2 2
1 2 3
a

EJEMPLOS
Encontrar la Longitud de la curva C en los siguientes casos:
2t

a. C : f t   e cos 3t , e sen3t
2t
 , 0t 


f t   e2t cos 3t , e2t sen3t  
f 't    2e 2t cos 3t  3e 2t sen3t;2e 2t sen3t  3e 2t cos 3t 
L
0

 2e 2t
 
2
cos 3t  3e 2t sen3t   2e 2t sen3t  3e 2t cos 3t dt 2


 e 2t 
L

e  4tt
13 cos 3t  13sen 3t dt  13 0 e dt  13  2
2 2

2t   13 1  e 2 
0  2
 0 

 t cos u  
b. f t   
t senu

0 u
du , 0 u
du  , 0  t 
 2

 t cos u   cos t sent 


f t    
t senu
du, du  f 't    , 
0 u 0 u
  t t 
 2 2  
 cos t   sent  cos 2 t sen 2 t 
 
dt  ln t  02  ln  
1
L 2
 t    t  dt   2  2 dt   2
1
    1 t2 t 1 t 2

c. f t   cost , sent , t  , 0  t  2

f t   cost , sent , t  f 't    sent , cos t ,1

L
0
2
 sent 2  cos t 2  1dt  0
2
 
2
2dt  2 t 0  2 2

56
Borrador del Módulo de Matemática II

ECUACIONES DIFERENCIALES

DEFINICIÓN: Una ecuación diferencia es la que contiene derivadas o


diferenciales de una función incógnita.
Ejemplos: y´ 4 x  7
y´dx  ln xdy  0
my' '  mg  ky'

 El orden de una ecuación diferencial ordinaria, está dado por el orden de la

región derivada.

 El grado de una ecuación diferencial ordinaria, está dado por el exponente de

mayor orden de su derivada.

EJEMPLOS:

d 2 y dy
1. e
x
 1 2 do orden, 1er grado
dx 2 dx
2
d2y
4
 dy 
2.  2   2   xy  0
do
3er orden, 2 grado
 dx   dx 

3.1. ECUACIONES DIFERENCIALES ORDINARIAS DE PRIMER ORDEN


Y DE PRIMER GRADO
A las ecuaciones diferenciales ordinarias de primer orden y de primer grado lo
representamos en la forma: F ( x, y, y´)  0
De la ecuación diferencial F ( x, y, y´)  0 despejamos y ` , es decir obtendremos
una ecuación de la forma y´ g ( x, y) .

3.2. `ECUACIONES DIFERENCIALES ORDINARIAS DE VARIABLE


SEPARABLE
Si la ecuación diferencial de primer orden y de primer grado y´ g ( x, y) podemos
expresar en la forma: M ( x)dx  N ( y)dy  0 (1)

57
Borrador del Módulo de Matemática II

Donde M es una función solo de xy N es una función solo de y , entonces la


ecuación (1) se denomina ecuación diferencial ordinaria de variable separable, y

su solución es por integración directa, es decir:  M ( x)dx   N ( y)dy  C


Donde C es la constante de integración.
EJEMPLOS

1. y 2
 xy 2 
dy
dx
 x2  x2 y  0

y 2
 
 xy 2 dy  x 2  x 2 y dx  0 
y 2 1  x dy  x 2 1  y dx  0  1  x 1  y 

x 2 dx y2 x2 1  1 y 2  1 1
 x  1  1  y dy  C
   x 1 dx   1  y dy  C
x  1x  1 dx  1  y 1  y  dy  C
 x 1  1 y
  x  1dx   1  y dy  C
1 2 1
x  x  ln 1  x  y  y 2  ln 1  y  0
2 2
1 2 1 2 1 x
Sol: x  y  y  x  ln 0
2 2 1 y

2. e x senydx  (1  e x )senytgydy  0 y  60º , x  3

ex
dx  tgydy  0  seny(1  e x )
(1  e x )

ex 1 e x 
 (1  e x ) dx   tgydy  C  ln(1  e )  ln(cos y)  C  ln cos y   ln(k )
x

1 e x 1 e3
Por inyectividad  k  Utilizando las condiciones tenemos: k
cos y cos 60º

1 ex
Reemplazando en  1  e 3   k Sol: 2(1  e ) 
3

cos y


3. 1  y 2 dx  y  1  y 2 1  x 2 2 dy  3

dx

y  1  y2 
1  x 
3
2 2 
1  y2
dy

58
Borrador del Módulo de Matemática II

dx y  1 y 2  dy  C
 dy  
1  y 
1  x 
3 2
2 2

dx
  ; x  tg  dx  sec2 d
1  x 
3
2 2

sec 2 d sec 2 d d x


 
sec 

sec 
  cos d  sen  C  C
1  tg  
3 3
2 2 1 x 2

x
sen 
1 x 2

y  1 y2  y

1
 1 1
   1 y2
dy 
  1 y 2
dy   1  y 2 2 dy 
2
ln(1  y 2 )  
1  y2
dy

y  tg  dx  sec2 d

sec 2 d sec 2 d


 1  tg 2

sec 
  sec d  ln(sec   tg )

y  1 y2 1 
  1 y 2
dy  ln(1  y 2 )  ln(sec  tg )
2

y  1 y2 1 
  1 y 2
dy   ln(1  y 2 )  ln( 1  y 2  y)
 2

(1  y  y 2 )dx  x( x 2  4)dy  0

1 1
dx  dy  0
x( x  4)
2
(1  y  y 2 )
1 1
 x( x 2
 4)
dx  
(1  y  y 2 )
dy  C

59
Borrador del Módulo de Matemática II

1
  x( x 2
 4)
dx

1 1 A B C
   
x( x  4)
2
x( x  2)( x  2) x ( x  2) ( x  2)

1 A( x  2)( x  2)  Bx ( x  2)  Cx( x  2)

x( x  2)( x  2) x( x  2)( x  2)

1  A( x  2)( x  2)  Bx( x  2)  Cx( x  2)

1
Si x=0 entonces 1  8B  B 
8

1
Si x=2 entonces 1  4 A  A 
4

1
Si x=-2 entonces 1  8C  C 
8

 1 1 1 
  
1 1 1 1
 x( x2  4) dx    x  ( x  2)  ( x  2) dx   4 ln( x)  8 ln( x  2)  8 ln( x  2)
 4 8 8
 
 

 x2  4 
 ln  
 x 2 
 
1 1 1
  (1  y  y ) dy    1
2
  1
dy  
1  3
2
dy
  y  y   1  
2
  y 
4   4 4  4
2  2 y 1  x 2  4  2  2 y 1
1
 
 (1  y  y 2 ) dy 
3
arctg 

 3 

  C ln
 x 
2

3
arctg 
 3 
  C

3.3. ECUACIONES DIFERENCIALES ORDINARIAS REDUCIBLES A


VARIABLE SEPARABLE
dy
La ecuación de la forma  f ax  by  c  donde a, b y c son constantes, no es
dx
de variable separada.
Para transformar a una ecuación de variable separable sustituimos z  ax  by  c ,
donde:

60
Borrador del Módulo de Matemática II

dy
 f ax  by  c  (1)
dx
z  ax  by  c
dz dy dy 1 dz a
 ab    , en (1)
dx dx dx b dx b
1 dz a
  f ( z )  La Cual es una E.C de variable separada
b dx b
1 a 
dz    f ( z ) dx  0
b b 
EJEMPLOS
Resolver las siguientes ecuaciones diferenciales.
1. xy 2 xy  y   a
z  y
z  xy  z  y  xy  y 
x
Reemplazando:
z 2   z  y   z2 z 2 dz
x 2  x   y  a  z  a  a  z 2 dz  axdx 
x   x   x x dx
z3 x2
z 2 dz  axdx  0   z 2 dz  a  xdx  C   a C
3 2
1 1
Sol:  x 3 y 3  ax 2  C
3 2
2. ln x  y dx  3xy 2 dy  0
3

ln x  y dx  3xy dy  0 ,


3 2
dividimos para: dx  ln x  y 3  3xy 2 y 
1
z  ln x  y 3  dz   3y 2 y Multiplicamos por x
x
xz   1  3xy 2 y   3xy 2 y   xz   1 Sustituimos en la E.D

 ln x  y dx  3xy dy  0


3 2

z  xz   1  0  z  1  xz   0  z  1  x
dz dz dx
 
dx 1 z x

 0   ln 1  z   ln x   ln C
dz dx dz dx
  0   
1 z x 1 z x

 
 
ln 1  ln x  y 3  ln x  ln K  ln 
x
  ln K
 ln x  y3  1 

x
Sol: K
ln x  y3  1

61
Borrador del Módulo de Matemática II

3. 1  xy cos xy dx  x 2 cos xydy  0


1  xy cos xy dx  x 2 cos xydy  0 Dividimos para: dx  1  xy cos xy  x 2 cos xyy   0

z  y
z  xy  z   y  y x  y  Sustituimos en la E.D
x
 1  xy cos xy  x 2 cos xyy   0

1  z cos z  cos zz x  z   0  1  xz  cos z  0  1  x


dx
cos z  0
dz

1 1
dx  cos zdz  0    dx   cos zdz  0  ln x  senz  C
x x

Sol: ln x  senxy  C

3.4. ECUACIONES DIFERENCIABLES ORDINARIAS HOMOGENEAS


Diremos que la función F x, y  es homogénea de grado K si solo si cumple que:

f (x, y)  k f ( x, y)
DEFINICIÓN: Diremos que la ecuación de primer orden y de primer grado de la
forma M ( x, y)dx  N ( x, y)dy  0 es homogénea si M y N son funciones
homogéneas del mismo grado en X e Y .
EJEMPLOS

1. ( x  y )dx  ( x y)dy  0
3 3 2

 M ( x, y)  x3  y 3  M (x, y)  (x) 3  (y) 3  M (x, y)  3 x3  3 y 3

M (x, y)  3 ( x3  y 3 )  M (x, y)  3 M ( x, y) La función M es

homogénea de grado 3

 N ( x, y)  x 2 y  N (x, y)  (x) 2 (y)  N (x, y) 

 2 x 2 y  N ( x, y) Es homogénea de grado 3

dy
2. x  y  2 xe y x

dx
Reescribir la ecuación para poder sacar los diferenciales.

( y  2 xe y x )dx  xdy  0

62
Borrador del Módulo de Matemática II

x
 M ( x, y)  ý  2 xe y x
 M (x, y)  y  2xe x
 M (x, y)   ( y  2 xe y x )

M (x, y)  M ( x, y) La función M es homogénea de Grado 1.

 N ( x, y)   x  N (x, y)   x   ( x)  N (x, y)  N ( x, y) La función

N es homogénea de Grado 1.

Para resolver una ecuación diferencial ordinaria homogénea


M ( x, y)dx  N ( x, y)dy  0  sustituimos: y  ux o´ x  uy y lo diferenciamos

dy  udx  xdu o´ dx  udy  ydu , convirtiéndose en una ecuación de variable


separable.
EJERCICIOS DE APLICACIÓN
Resolver las siguientes ecuaciones diferenciales:
1. ( x 2  3xy  y 2 )dx  x 2 dy  0

y  ux  dy  udx  xdu E.D.H con M y N homogéneas de Grado 2.

( x 3  3xu  u 2 x 2 )dx  x 2 (udx  xdu )  0  x 2 (1  3u  u 2 )dx  ux 2 dx  x 3 du  0

x 2 (1  3u  u 2 )dx  ux 2 dx  x 3 du  0  x 2 (1  3u  u 2  u)dx  x 3 du  0
1 1 1 1 1
x
dx 
1  2u  u 2
du  0   x dx   1  2u  u 2
du  c  ln x 
1 u
c

x
Sol: ln x  c
x y
dy
2. x  y  y2  x2
dx

xdy  ( y  y 2  x 2 )dx

x  uy  dx  udy  ydu

uydy  ( y  y 2  u 2 y 2 )udy  ydu )  y  y 2  u 2 y 2 )udy  ydu  uydy  0

( y  y 1  u 2 )udy  ( y  y 1  u 2 ) ydy  uydy  0

uy(1  1  u 2  1)dy  y 2 (1  1  u 2 )du  0

1 1 1 u2 1 1 1 u2
y
dy 
u 1 u2
du  0   y  u 1  u 2 du  c
dy 

63
Borrador del Módulo de Matemática II

1 1 u2
ln y   . du  c  ln y  ln u  arcsen(u)  c
u 1 u2 u 1 u2
x x
Sol: ln y  ln( )  arcsen( )  c
y y

3. ( x  y ln y  y ln x)dx  x(ln y  ln x)dy  0

x y
Reescribiendo: [ x  y ln( )]  x ln( )dy  0
y x

y  ux  dy  udx  xdu

1 1
[ x  ux ln( )]dx  ( x ln u )(udx  xdu )  0  [ x  ux ln( )]dx  ux ln udx  x 2 ln udu  0
u u
1
( x  ux ln u  ux ln u)dx  x 2 ln udu  0  xdx  x 2 ln udu  0  dx  ln udu  0
x
1 1
x
dx  ln udu  0   xdx   ln udu  c w  ln u dv  du

1
ln x  u ln u  u  c dw  du vw
u
y y y
Sol: ln x  ln   c
x x x
y y
4. dy  (  csc 2 )dx
x x
y y
[  csc 2 ]dx  dy  0
x x
y  ux  dy  udx  xdu

(u  csc2 u)dx  udx  xdu  0  (u  csc2 u  u)dx  udx  xdu  0

1 1 1
 csc2udx  xdu  0  dx  2
du  0  dx  sen 2udu  0
x csc u x
1 1  cos 2u 1 1
 x dx   sen udu  c  ln x   2 du  c  ln x  2 u  4 sen2u  0
2

1y 1 y
Sol: ln x   sen2( )  c
2x 4 x
5. 2 xy´(x 2  y 2 )  y( y 2  2 x 2 )

dy 2
2x ( x  y 2 )  y( y 2  2 x 2 )  y( y 2  2 x 2 )dy  2 x( x 2  y 2 )dx  0
dx

64
Borrador del Módulo de Matemática II

y  ux  dy  udx  xdu

ux(u 2 x 2  2 x 2 )(udx  xdu )  2 x( x 2  u 2 x 2 )dx  0

u 4 x3dx  2ux3dx  2u 2 x3dx  2ux 4 du  2x3dx  2u 2 x3dx  0   u 3 x3dx  2 x 4 (1  u 2 )du  0

1 2(1  u 2 ) 1 1 1
dx  3
du  0  ln x  2 3 du  2 du  0  ln x  2  2 ln u  c
x u u u u
2 2
1 y x
ln x  2  2 ln u  c  Sol: ln( )  2  c
u x y

3.5. ECUACIONES DIFERENCIALES REDUCIBLES A HOMOGÉNEAS


ax  by  c
La ecuación de la forma y` f ( )  no es homogénea por la existencia
a ' x  b' y  c '
de las constantes c y c’, estas constantes se pueden eliminar mediante una
traslación, transformando la ecuación  en una ecuación diferenciable
homogénea, para esto consideremos las ecuaciones: L1 : ax  by  c  0 y
L2 : a' x  b' y  c'  0

Si L1  L2  p(h, k )  L1  L2 y sustituyendo x  z  h e y  w  k y con sus

respectivos diferenciales en la ecuación  se transforma en una ecuación


homogénea.- Luego se utiliza z  uw y su diferencial para transformar a la
ecuación en una de variable separable.

NOTA: Para casos especiales se utiliza y  z .


Ejemplos
Resolver las siguientes ecuaciones diferenciales:
dy x  y  1
1. 
dx x  y  1

( x  y 1)dx  ( x  y  z)dy  0

L1 : x  y  1  0  x  y  1
  P(0,1)
L2 : x  y  z  0 x  y  1

x  z  0 dx  dz
  
 y  w 1 dy  dw
( z  w  1 1)dz  ( z  w 1  1)dw  0  ( z  w)dz  ( z  w)dw  0

z  uw dz  udw  wdu

65
Borrador del Módulo de Matemática II

(uw  w)(udw  wdu)  (uw  w)dw  0  (u 2 w  w)dw  w2 (u  1)du  0

w(u 2  1)dw  w2 (u  1)du  0  w2 (u 2  1)

1 u 1 1 u 1 u 1

w u2 1
0   w dw   u 2
1
du  c  ln w   2
u 1
du   2 du  c
u 1
 x  x
ln w  ln u 2  1  arctg (u)  c Sol: ln (y - 1)  1   arctg ( )c
 ( y  1) 2
 y 1

2. x  4 y  9dx  4x  y  2  0
L1 : x  4 y  9  0 x  4 y  9
  P(1,2)
L2 : 4 x  y  2  0 4 x  y  2

x  z  1 dx  dz
  
y  w  2 dy  dw
( z  1  4w  8  9)dz  (4 z  4  w  2  2)dw  0  ( z  4w)dz  (4 z  w)dw  0

z  uw dz  udw  wdu

(uw  4w)(udw  wdu)  (4uw  w)dw  0  (u 2 w  w)dw  w 2 (u  4)du  0

w(u 2  1)dw  w2 (u  1)du  0  w2 (u 2  1)

1 u4 1 u du udu du

w u2 1
0  w dw   u 2
1
du  4 2
u 1
 c  ln w   2
u 1
 4 2
u 1
c

ln u 2  1  4arctg (u )  C x  12   y  22   4arctg ( x  1 )  c


1
ln w  Sol: ln 
2   y2

3. 4 xy 2 dx  (3x 2  1)dy  0

y  z  dy  z dz

4 xz 2 dx  (3x 2 z 2 1  z 1 )dz  0

4 xz 2 grado 2  1 2  1   1

3x 2 z 2 1 grado 2  1   2 reemplazar en 


z 1 grado   1

4x  6x2 2
4 xz dx  (6 x z  2 z )dz  0 
4 2 5 3
dx  ( 5  3 )dz  0 
z4 z z
4 xzdx  (6 x  2 z )dz  0 x  uz dx  udz  zdu
2 2

4(uz) z(udz  zdu)  (6(uz) 2  2 z 2 )dz  0  4u 2 z 2 dz  4uz 3du  (6u 2 z 2  2 z 2 )dz  0


4uz 3du  (2 z 2  2u 2 z 2 )dz  0  4uz 3 du  2 z 2 (1  u 2 )dz  0 
 2z 3 1  u 2 
66
Borrador del Módulo de Matemática II

4u 2 u 1
du  dz  0  4 du  2 dz  c
1 u 2
z 1 u 2
z
 z2   z2 
 2 ln(1  u 2 )  2 ln z  ln k  ln    ln k   2
k
 1  u 2    1  u 2  
2

1 1 x x
como: y  z 2  z y u  x y
z 2
y z 1
y

1
Sol: k
y (1  x 2 y ) 2

4. ( y 4  3x 2 )dy   xydx

y  z  dy  z dz

( z 4  3x 2 )z 1dz   xz  dx  xz  dx  (z 5 1  3x 2 z 1 )dz  0

xz  grado   1 5 1    1

z 5 1 grado 5  1 
1
reemplazar en 
2

3x 2 z  1 grado   1
1 1 3 3 1 1 1 3 3x 2
xz 2 dx  ( z 2  x 2 z 2 )dz  0  xz 2 dx  ( z 2  1 )dz  0
2 2 2 z 2
1 2
xzdz  ( z  3x 2 )dz  0 x  ux dx  xdu  udx
2
1 1
u 2 z 2 dz  uz 3du  ( z 2  3u 2 z 2 )dz  0  uz 3du  ( z 2  3u 2 z 2  2u 2 z 2 )dz  0
2 2
1 2 u 1 u 1
uz 3du  z (1  u 2 )dz  0  du  dz  0   1  u du   2 z dz  c
2 1 u 2

2z  2

1 1  z 12   z 12 
 ln(1  u )  ln z  ln k  ln 
2
  ln k   k
 (1  u 2 ) 2   (1  u 2 ) 2 
1 1
2 2

y
Sol: 1
k
 x  2 2
1  4 
 y 

5. y  cos xdx  (2 y  senx)dy  0

z  senx  dz  cos xdx

67
Borrador del Módulo de Matemática II

ydz  (2 y  z)dy  0 z  uy dz  udy  ydu

2
y(udy  ydu )  (2 y  uy)dy  0  y 2 du  2 ydy  0  du  dy  0
y

2 sex
 du   y dy  c  u  2 ln y  c Sol:  2 ln y  k
y

dy 2 x  3 y  1
6. 
dx 3x  2 y  5

(3x  2 y  5)dy  2 x  3 y  1dx

L1 : 2 x  3 y  1  0 2 x  3 y  1
  P(1,1)
L2 : 3x  2 y  5  0 3x  2 y  5

x  z  1 dx  dz
  
y  w 1 dy  dw
2( z  1)  3(w 1)  1dz  3( z  1)  2(w 1)  5dw  0
(2 z  3w)dz  (3z  2w)dw  0
z  uw dz  udw  wdu
(2(uw)  3w)  (udw  wdu)  (3uw  2w)dw  0

2u 2 wdw  2uw2 du  3uwdw  3w2 du  (3uw  2w)dw  0

w 2 (2u  3)du  w(2u 2  6u  2)dw  0  w 2 (2u 2  6u  2)


2u  3 1
 2u 2
 6u  2
du   dw  c
w
2 ln w  ln(u 2  1)  3arctgu  c



Sol: ln 
 y  1
2
  x 1
  3arctg    C
  y  1x  1  1 

 y  1

3.6. ECUACIONES DIFERENCIALES EXACTAS


DIFERENCIAL TOTAL: Si f : R 2  R , es una función diferenciable en un punto

x, y   R 2 , entonces la diferencial total de f es la función df y está definido por:

f x, y  f x, y 
f x, y   dx   fxdx  fydy
x y

68
Borrador del Módulo de Matemática II

DIFERENCIAL EXACTA: Una expresión de la forma M x, y dx  N x, y dy  0 se

denomina exacta si existe una función f : D  R 2  R / df x, y   M x, y dx  N x, y dy

Es decir, que toda expresión que es la diferencial toral de alguna función de x e

y se llama diferencial exacta.

DEFINICIÓN: Consideremos la ecuación diferencial M x, y dx  N x, y dy  0 

Si existe una función z  f x, y  / f x  M x, y  y f y  N x, y  diremos que la

ecuación  es una ecuación diferencia exacta.


TEOREMA: La condición necesaria y suficiente para que una Ecuación
Diferencial M x, y dx  N x, y dy  0 sea exacta es que: M y  N x .

EJEMPLOS:
Resolver las siguientes ecuaciones diferenciales:
1. 2xy 2
  
 2 y dx  2 x 2 y  2 x dy  0

M x, y   2 xy 2  2 y  M y  4 xy  2 M y  NX

M x, y   2 x 2 y  2 x  N x  4 xy  2

Como la ecuación es exacta debe existir f x, y  / f x  M x, y  y f y  N x, y 

f x  M x, y   f x  2 xy 2  2 y Integramos con respecto a x

 
f x, y    2 xy 2  2 y dx  g  y   f ( x, y)  x 2 y 2  2 xy  g ( y) 1

Derivamos con respecto a f y

f y  2 x 2 y  2 x  g ' ( y) , como f y  2 x 2 y  2 x al sustituir se tiene:

2 x 2 y  2 x  2 x 2 y  2 x  g ( y)  g ' ( y)  0  g ( y)  C Remplazar en 1

f ( x, y)  x 2 y 2  2 xy  C Sol: x 2 y 2  2 xy  K

2. e seny  2 ysenx dx  e


x x

cos y  2 cos x dy  0

M x, y   e x seny  2 ysenx  M y  e cos y  2senx M y  NX


x

M x, y   e x cos y  2 cos x  N x  e x cos y  2senx

Como la ecuación es exacta debe existir f x, y  / f x  M x, y  y f y  N x, y 

f x  M x, y   f x  e x seny  2 ysenx Integramos con respecto a x

69
Borrador del Módulo de Matemática II

 
f x, y    e x seny  2 ysenx dx  g  y   f ( x, y)  e x seny  2 y cos x  g ( y) 1

Derivamos con respecto a f y

f y  e x cos y  2 cos x  g ' ( y) , como f y  e x cos y  2 cos x al sustituir se tiene:

e x cos y  2 cos x  e x cos y  2 cos x  g ( y)  g ' ( y)  0  g ( y)  C Remplazar en 1

f ( x, y)  e x seny  2 y cos x  C Sol: e x seny  2 y cos x  K

 1 1  1 x 
3.    dx     2 dy  0
x y
 x2  y2 x y   x2  y2 y y 
   

xy 1
M x, y  
x 1 1
   My   M y  NX
x 
3
x2  y2 x y y2
2
y 2 2

M x, y  
y 1 x xy 1
   Nx   
y y2
x 
3
x2  y2 y2
2
y 2 2

Como la ecuación es exacta debe existir f x, y  / f x  M x, y  y f y  N x, y 

f x  M x, y  
x 1 1
fx    Integramos con respecto a x
x y
2 2 x y

 1 1
f x, y    
x x
  dx  g  y   f ( x, y )  x  y  2 ln x   g ( y )
2 2
1
 x2  y2 x y  y
 
Derivamos con respecto a f y

y x
fy    g ' ( y ) , como f y  2 x 2 y  2 x al sustituir se tiene:
x2  y2 y2

y 1 x 1 1
  2  2 x 2 y  2 x  g ( y )  g ' ( y )   g ( y )   dy  C
x2  y2 y y y y

g ( y)  ln y  C Remplazar en 1
x x
f ( x, y)  x 2  y 2  2 ln x  C Sol: x 2  y 2  2 ln x  K
y y

 y   x 
4.   arcty dx    arctx dy  0
1 x  1 y
2 2

1 1
M x, y  
y
 arcty  My   M y  NX
1 x2 1 x 1 y2
2

70
Borrador del Módulo de Matemática II

1
M x, y  
x
 arctx  Nx 
1 y2 1 x2
Como la ecuación es exacta debe existir f x, y  / f x  M x, y  y f y  N x, y 

f x  M x, y  
y
fx   arcty Integramos con respecto a x
1 x2
 y 
f x, y      arcty dx  g  y   f ( x, y)  yarctx  xarcty  g  y  1
 1  x 2

Derivamos con respecto a f y

x x
fy   arctx  g ' ( y ) , como f y   arctx al sustituir se tiene:
1 y 2
1 y2

 arctx  g ( y)  g ' ( y)  o  g ( y)  C Remplazar en 1


x x
 arctx 
1 y 2
1 y2
f ( x, y)  yarctx  xarcty  C Sol: yarctx  xarcty  K

3.7. ECUACIONES DIFERENCIALES REDUCIBLES A EXACTAS


Consideremos la Ecuación Diferencial M x, y dx  Nxx, y dy  0  , si la ecuación
 , no es exacta, se puede transformar en exacta multiplicamos a la ecuación por:
ux, y  conocido como factor integrante.

Entonces ux, y M x, y dx  ux, y Nxx, y dy  0 , es una ecuación exacta y se

u x, y M x, y  u x, y Nxx, y 


cumple que  .
y x
Para determinar el factor integrante consideremos los siguientes casos:
f  x dx
1 CASO: Si u es una función solo de x , ux   e  , f x   My  Nx  .
1
N
g  y dy
2 CASO: Si u es una función solo de y , u y   e  , g  y    1 My  Nx .
M
3 CASO: En ciertos ejercicios el factor integrante está dado por un producto de
funciones f x  y g  y  , es decir ux, y   f x .g  y  que debe satisfacer la condición

que My  Nx  N f x   M g  y  se debe verificar por inspección.


f x  gy

4 CASO: Para ciertos ejercicios ux, y   x m y n , donde m y n se determinan


mediante la condición necesaria y suficiente de las ecuaciones diferenciales
exactas.

71
Borrador del Módulo de Matemática II

EJERCICIOS
Resolver las siguientes ecuaciones diferenciales:
1. 1  x 2 y dx  x 2  y  x dy  0

M ( x, y)  1  x 2 y  My   x 2 My  Nx E.D no Exacta.

N ( x, y)  x 2 y  x 3  Nx  2 xy  3x 2

My  Nx   x 2  2 xy  3x 2  2 xx  y 

f x  
1
My  Nx   f x   2 x2x  y   f x    2  f x  solo depende de
N  x x  y  x

x
2
 u x   e 
f  x dx
u x   e   ux   e 2 ln x  ux   e ln x  u x  
 dx 2 1
x
x2
Multiplicando la ecuación diferencial por u x  nos queda:

1  x y dx  x  y  xdy  0
2 2

M x, y  
1
y  M y  1
x2
N ( x, y)  y  x  N x  1 M y  N x Es E.D. Exacta

f ( x, y) / f x  M ( x, y)  f y  N ( x, y)

f y  N ( x, y)  f y  y  x

y2
f ( x, y)    y  x dy  g ( x)  f ( x, y )   xy  g ( x) 1
2
1 1 1
f x   y  g ' ( x)  2
 y   y  g ( x)  g ' ( x)  2  g ( x)   2 dx  C
x x x

g ( x)    C Remplazar en 1
1
x
y2 1 y2 1
f ( x, y)   xy   C Sol:  xy   K
2 x 2 x
 y
2.  dx  y  ln x dy  0
2
 
 
x

M  x, y  
y 1
 My 
x x
1
N ( x, y)  y 2  ln x  Nx   My  Nx E.D no Exacta.
x

72
Borrador del Módulo de Matemática II

2
M y  N x   1  1  2  g  y   1 M y  N x   xy  2  g  y  Depende sólo de y.
x x x M y
x
2
g  y dy   dy
u y   e 
1
 e y  e 2 ln y  e ln  y   y 2  2
2

 1  ln x 
Multiplicando la E.D. por u  y  nos da:  dx   y  2 dy  0
 xy   y 

M  x, y  
1 1
 My   2
xy xy
ln x 1
N ( x, y )  y   Nx   M y  N x Es E.D. Exacta
y2 xy 2
f ( x, y) / f x  M ( x, y)  f y  N ( x, y)

1
f x  M ( x, y)  f x 
xy

 1
 h( y ) 1
ln x
f ( x, y)    dy  h( y )  f ( x, y) 
 xy  y

ln x ln x ln x
fy  
y 2
 g ' ( y )  y  2   2  g ' ( y )  y  g ' ( y) 
y y  ydx  g ( y)
y  g ( y) Remplazar en 1
1 2
2
y2 1 y2 1
f ( x, y)   xy  y 2  C  Sol:  xy  y 2  K
2 2 2 2
3. xy  x 2
  
y  y 3 dx  x 2  1y 2 dy  0


M x, y   xy  x 2 y  y 3   M y  x  x 2  3y 2

N ( x, y)  x 2  1y 2  N x  2x M y  N x  Es E.D. No.Exacta

M y  N x   x 2  3y 2  x

ux, y   f x   g  y 
f ' x  g'y f ' x  g' y
M  Nx  N M N  ; M 
f x  gy f x  gy
y

x 2  3 y 2  x  ( x 2  2 y 2 )  ( xy  x 2 y  y 3 )

x 2  3 y 2  x  x 2  2y 2  xy  x 2 y  y 3 

73
Borrador del Módulo de Matemática II

x 2  3 y 2  x  (   ) x 2  (2  y) y 2  xy

    1

2  y  3 Encontramos  y  por cualquier método de sistema de ecuaciones
   1

1
 ;  2
y

 f ' x   f ' x 
 f x   2   f x  dx   2dx  ln f x   2 e f  x   e 2
Entonces:    
g'y 1 g'y
dy   dy ln g  y   ln y  g  y   y
1
  
 g  y  y  g  y  y

 f x   e 2 x
 Por lo tanto ux, y   ye 2 x es el factor integrante
 gy  y
Multiplicamos por E.D.
xy e2 2x
  
 x 2 y 2 e 2 x  y 4 e 2 x dx  x 2 ye 2 x  1y 3 e 2 x dy  0

M x, y   xy 2 e 2 x  x 2 y 2 e 2 x  y 4 e 2 x  M y  2 xye 2 x  2 x 2 ye 2 x  4 y 3e2 x

N ( x, y)  x 2 ye 2 x  1y 3e 2 x  N x  2 xye 2 x  2 x 2 ye 2 x  4 y 3e2 x M y  N x Es E.D.

Exacta
f ( x, y) / f x  M ( x, y)  f y  N ( x, y)

f y  N ( x, y)  f y  x ye  1y e
2 2x 3 2x

 
f ( x, y)   x 2 ye 2 x  1y 3 e 2 x dy  g ( x)  f ( x, y)  x y e  y e  g ( x) 1
1 2 2 2x 1 4 2x
2 2

fx 
y2
2
 
2 xe 2 x  2 x 2 e 2 x  y 4 e 2 x  g ' ( x)

y2
2
 
2 xe 2 x  2 x 2 e 2 x  y 4 e 2 x 
y2
2
 
2 xe 2 x  2 x 2 e 2 x  y 4 e 2 x  g ' ( x)

0  g ' ( x)  C  g (x) Remplazar en (1)

1 2 2 2x 1 4 2x 1 2 2 2x 1 4 2x
f ( x, y)  x y e  y e C  Sol: x y e  y e K
2 2 2 2
4. 2 y dx  x  xy 3 dy  0
M x, y   2 y  My 2

N ( x, y)  x  xy 3  N x  1  y 3 My  Nx E.D no Exacta.

74
Borrador del Módulo de Matemática II

ux, y   x m y n Multiplicando el factor integrante a los miembros de la E.D.

 
nos da: 2 x m y n1dx  x m1 y n  x m1 y n3 dy  0 1

M x, y   2 x m y n1  M y  2n  1x m y n

N ( x, y)  x m1 y n  x m1 y n3  N x  m  1x m y n  m  1x m y n3

Como al multiplicar el factor integrante por los miembros de la E.D la convierte en


exacta, se debe cumplir que:
n  1
M y  N x  2n  1x m y n  m  1x m y n  m  1x m y n3  2n  1  m  1  
m  1

Reemplazando en 1 2 x 1  y 1  y 2 dy  0 
M x, y   2 x 1  M y  0

N ( x, y)  y 1  y 2  Nx  0 M y  N x E.D.Exacta

f ( x, y) / f x  M ( x, y)  f y  N ( x, y)

2
f x  M ( x, y)  f x 
x
2
f ( x, y )    dy  g ( y )  f ( x, y)  2 ln x  g ( y) 2
 x

 y 
1
f y  g ' ( y)  y 1  y 2   g ' ( y)   y 2 dy  g ( y)

y3
g ( y )   ln y   C Remplazar en 2
3
y3 x2 y3
f ( x, y)  2 ln x  ln y  C  Sol: ln  K
3 y 3

3.8. ECUACIONES DIFERENCIALES LINEALES DE PRIMER ORDEN


A una ecuación de la forma y  px y  qx  , llamaremos ecuación diferencial
lineal de primer orden en la variable y .

1. Si qx   0 , entonces y  px y  qx  es una ecuación diferencial lineal

 p  x dx
homogénea de variable separable y su función es y  ke  .

2. Si qx   0 , la ecuación diferencial la llamaremos Ecuación Diferencial lineal no

homogénea esta no es exacta.

75
Borrador del Módulo de Matemática II

p  x dx
3. Si llamamos un factor integrante I x   e  , y lo multiplicamos a los

miembros de la ecuación diferencial, su solución general es de la forma:

 p  x dx   p  x dx qx dx  C 


ye    e 

EJEMPLOS
Resolver las siguientes Ecuaciones Diferenciales:
1. y  2 y  x 2  2 x

p x   2 ; q x   x 2  2 x

ye 
 p  x dx 
  e    
 p  x dx qx dx  C   y  e  2 dx  e  2 dx x 2  2 x dx  C 

  
y  e 2 x  e 2 x x 2  2 x dx  C 
e2x
  x e dx   2xe dx u  x 2 ; du  2 x ; dv  e 2 x ; v 
2 2x 2x

1 2 2x 1
x e   xe 2 x   2 xe 2 x dx  x 2 e 2 x   xe 2 x dx
2 2

e2x
u  x ; du  dx ; dv  e 2 x ; v 
2

1 2 2x 1 2 2x 1 2x 1 1 1
x e  x e   e dx  x 2 e 2 x  x 2 e 2 x  e 2 x
2 2 2 2 2 4

1 1 1  1 2 1 2 1
y  e 2 x  x 2 e 2 x  x 2 e 2 x  e 2 x  C  Sol: y  x  x   Ce 2 x
2 2 4  2 2 4
1
2. y 
xseny  2sen2 y

dy 1 dx
   xseny  2sen2 y  x  xseny  2sen2 y
dx xseny  2sen2 y dy
p y   seny ; q y   2sen2 y
 p  y dy   p  y dy q y dx  C   x  e  senydy  e  senydy 2sen2 y dy  C 
xe   e   

x  e  cos y 2 e cos y sen2 y dy  C 
  e sen2 y dy  C  2 e
cos y cos y
seny.cos ydy

76
Borrador del Módulo de Matemática II

u  cos y ; du  senydy ; dv  e cos y senydy ; v  e cos y

  
2  cos y.e cos y   e cos y senydy  2  cos y.e cos y  e cos y 

x  4e  cos y  cos y.e cos y  e cos y  Sol: x  4 cos y  4  Ce  cos y

3.9. ECUACIONES DIFERENCIALES BERNOULLI


Una ecuación diferencial de Bernoulli es de la forma: y´ p( x) y  q( x) y´´ ; Para n  1
transformar una Ecuación Diferencial de Bernoulli a una ecuación diferencial lineal
se procede de la siguiente manera:
1. A la Ecuación Diferencial se multiplica por y n , es decir.

y  n y´ p( x) y1n  q( x)

2. Luego se multiplica. (1  n) , es decir (1  n) y  n y´(1  n) p( x) y1n  (1  n)q( x)

3. Sustituir. z  y1n  z´ (1  n) y  n y´ ,es decir: z´(1  n) p( x) z  (1  n)q( x)

4. Al reemplazar los resultados del tercer paso en la Ecuación Diferencial del

segundo paso se tiene: z´(1  n) p( x) z  (1  n)q( x)  q´ Es una Ecuación

Diferencial en z.
EJERCICIOS DE APLICACION
Resuelva las siguientes Ecuaciones Diferenciales:
1. 2 xy´2 y  xy 3

y y3
2 xy´2 y  xy  y ' 
3
  y3
x 2

y 2 1 2 y 2
y 3 y´    2 y 3 y´  1 1
x 2 x

z  y 2  z´ 2 y 3 y´

  2 xdx 
 
2 dx

2z
1  x dx  c  z  eln x eln x 1  xdx  c
2 2
z´ 1 z  e x
e
x  

1  1 1
z  x 2   c   z  x  cx 2  z  2  2  x  cx 2
x  y y

77
Borrador del Módulo de Matemática II

1
Sol: y  
x  cx 2


2. y 2 y 6  x 2 y´ 2 x 
y8 x2 y 2
 2 xx´ y 2 x 2  y 8 1
2x dy 2x
y´    x´ 
y x y
8 2 2
dx y  x y
8 2 2
2x 2x

z  x 2  z´ 2 xx´

z´ y 2 z  y 8  z  e  y dy e  y dy y 8 dy  c  z  e  3  e 3 y 8 dy  c 
3 3

   
2 2 y y

  
 
y3 y3 y3

y e uy  dv   y e dx  v  e
2 3 6 6 2 3 3
y dy

y3 y3 y3 y3 y3 y3 y3
6
y e 3
 6 y e dy  y e 3 3 6 3
 6y e 2 3
 18 y e dy2 3
u  y ;  dv   y e dx  v  e
3 2 3 3

y3 y3 y3
6
y e 3
 6y e 2 3
 18e 3

y3
y3  6 y  
3 3 3
y y

z  y 6
 6 y 2
 18  ce 3
 z  x2
ze  y e  6 y e  18e  c  
3 3 2 3 3

 
y3

Sol: x  y  6 y  18  ce
2 6 2 3

3. 3xdy  y 1  xsenx  3 y senx dx


3
 

3xy´ y 1  xsenx  3 y 3 senx dx  3xy´ y  ysenx  3 y 3 senx 
y ysenx 3 y 3 senx 1  xsenx 3 ysenx
y´    y 4 y´ y  
senx
1
3x 3 3x 3x 3 x

z  y 3  dz  3 y 4 y´

(1  xsenx) 3senx 
1 xsenx
dx   1 xsexn dx  3senx  
z´ z  ze  e  dx  c 
x x
x x   x  
  senx   cos x  senx 
z  e ln xcos x  e ln xcox  dx  c   e dx   e  cos x senxdx   e
 cos x
x
  x    x 

3 3 ce cos x
Sol: y  
x x
dx y x( x  ln x)
4. x  
dy ln x y 2 ln x

78
Borrador del Módulo de Matemática II

y x( x  ln x) y x  ln x y x  ln x
xy´  2
 y´   y 2  y 2 y´ 
ln x y ln x x ln x ln x x ln x ln x

z  y 3  dt  3 y 2 dy 1

 x ln x   x ln x  3( x  ln x)     3( x  ln x)  
3 dx 3 dx

ze  e    c Sol: z  e 3 ln(ln x ) e3 ln(ln x )    c
  ln x     ln x  

3.10. ECUACIONES DIFERENCIALES DE RICCATI


Una Ecuación Diferencial de Recata es de la forma: y´ p( x) y 2  q( x) y  r ( x) , esta
ecuación no se puede resolver por los métodos vistos hasta el momento para
resolver Ecuaciones Diferenciales Riccati se debe conocer una solución particular
y   (x) y a partir de esta haciendo y   ( x)  z, diferenciando y´ ´(x)  z´ y
reemplazando en la Ecuación Diferencial dada, más los respectivos procesos
algebraicos se llega a una ecuación de Bernoulli cuyo proceso para resolver ya lo
conocemos.
EJERCICIOS DE APLICACIÓN
Resolver las siguientes ecuaciones diferenciales
y2 y
1. y´  1
x2 x

y  x  z  y´ 1  z´

 x 2  2 xz  z 2  z
1  z´  2
  1   1
 x  x

2z z 2 z 3z z 2
1  z´ 1   2   z´  2  z 2
x x x x x

 z 2 z´ z 1   2 1 w  z 1  w´  z 2 z´ en1


3 1
x x
  dx   3  3 1  
3
 dx  1 
3
3w 1
w´   2  w  e x   e x   2 dx  c   w  e ln x   e ln x   2 dx  c 
x x   x     x  

w
1
x3

 
 x 2 1 

x 
2  dx  c

  w 
1
x3 
 xdx  c  
w 
1  x2 
x 3  2 

   c
 
1 1 1 1 c
w  Sol:   3
z yx yx 2x x
dy 2senx 1
2.  y 2 senx  sol : y 
dx cos 2 x cos x

79
Borrador del Módulo de Matemática II

y  sec x  z  y´ sec xtgx  z´

sec x.tgx  z´sec x  z  senx 


2 2senx
cos 2 x
2senx
sec x.tgx  z´ sec 2 x.senx  2senx.z. sec x  z 2 senx 
cos 2 x
senx 2 zsenx 2senx
sec x.tgx  z´ 2
  z 2 senx  0
cos x cos x cos 2 x
2 z.senx
sec x.tgx  z´  z 2 senx  sec x.tgx  0
cos x
z´2 z.tgx   z 2 senx  z 2  z 2 z´2tgx.z 1  senx

w  z 1  w´  z 2 z´
2 tgx.dx 
w´2w.tgx  senx  w  e  senx dx  c
 2  tgx.dx
 e

 
w  e 2 ln(cos x )  e 2 ln(cos x ) senx dx  c  w 
1
2
cos x

 cos 2 xsenx dx  c 
1  cos 3 x  cos x c 1
w   c  w     w
cos 2 x 3  3 2
cos x 1
y
cos x
1 cos c 3 cos 2 x
  Sol: y   sec x
y  sec x 3 cos 2 x c  cos 3 x

3.11. ECUACIONES DIFERENCIALES LAGRANJE Y CLAIROUTS


Las Ecuaciones Diferenciales de LaGrange y Clairouts son de la forma:
a) y  x. f ( y´) 1

Para resolver este tipo de ecuación de LaGrange se transforma en otra ecuación


lineal en x como función de p , haciendo y´ p  y  xf ( p)  g ( p) , diferenciando
se tiene dy  f ( p)dx  xf ´( p)dp  g´( p)dp , pero dy  pdx :
pdx  f ( p)dx  xf ´( p)dp  g´( p)dp

 p  f ( p)dx  xf ´( p)dp  g´( p)dp  dp

 p  f ( p) dx  xf ´( p)  g´( p)
dp

 p  f ( p)x´ xf ´( p)  g´( p)   p  f ( p)


xf ´( p) g´( p)
x´   E.D.L.1er .orden
p  f ( p) p  f ( p)

80
Borrador del Módulo de Matemática II

Cuya solución es: x   ( p, c) , la solución general de la ecuación 1 es:

 x   ( p, c )

 y   ( p, c). f ( p)  g ( p)
b) Las ecuaciones diferenciales de Clairouts son de la forma y  xy´ g ( y´) , para

resolver seguir procedimiento del literal a )

EJERCICIOS DE APLICACIÓN
Resolver las siguientes Ecuaciones Diferenciales
1. 2 y  xy´ y´ln( y´) 1

y´ p  dy  pdx  2 y  xp  p ln p 2

2dy  xdp  pdx  dp  ln p.dp  2 pdx  xdp  pdx  dp  ln p.dp

dx x ln p  1 x ln p  1
   x´ 
dp p p p p

 p    p  ln p  1 
dp dp
   ln p  ln p  1  
x  e  e  dp  c   x  e   e
ln p
 dp  c 
  p     p  

  ln p  1  
x  p    2
dp  c 
  p  

ln p dp dp dp 1
  2
dp   2 u  ln p  du  ;  dv   2  v  
p p p p p

 ln p dp dp   ln p 2 
x  p    2   2  c   x  p    c
 p p p   p p 

x  cp  2  ln p Reemplazar en 1  2 y   p ln p  2 p  cp 2  p ln p

 x  cp  ln p  2
cp 2 
y  p Sol:  y  p cp  1
2 
  2 

2. y  2 xy´seny´

y´ p  dy  pdx

y  2 xp  senp 1
dy  2 xdp  2 pdx  cos p.dp  pdx  2 xdp  2 pdx  cos p.dp

81
Borrador del Módulo de Matemática II

dx 2 x cos p 2x cos p
 pdx  2 xdp  cos p     x´ 
dp p p p p

 
2 dp
  2 dp
p  cos p   2  2  cos p  
xe p
 e   dp  c   x  e ln p  e ln p   dp  c
  p     p  

x
1  2  cos p 
2 
p  


dp  c  x  2    p. cos p dp  c
1

p   p   p

u  p  du  dp  dv   cos p.dp  v  senp


x
1
p 2
 p
1

 p.senp   senp.dp  c  x  2  p.senp  p. cos p  c

Reemplazar en 1
c cos p senp
x  2 
p2 p p
2 cos p 2c 2c 2 cos p
y  2senp    senp  y   senp 
p p p p

 senp cos p c
 x   p  p 2  y 2
Sol: 
2c 2 cos p
 y   senp 
 p p

3.12. ECUACIONES DIFERENCIALES DE ORDEN SUPERIOR


dmy
1 CASO: Las ecuaciones diferenciales de la forma  f (x) 1 la solución de
dx m

la ecuación 1 se obtiene por integración sucesiva.


d2y
2 CASO: La ecuación de la forma  g ( y )  2 , para la solución se toma en
dx 2
cuenta que:
d 2 y d  dy  dy´ dy´ dy dy´ y´dy´
    .  
dx 2
dx  dx  dx dy dx dx dy

Sustituyo en 2 se tiene


y´dy´
 g ( y)  y´dy´ g ( y)dy , integrando:
dy

y ´2
 y´dy´  g ( y)dy    g ( y )dy  c
2

y´ 2 g ( y)dy  c1  y   2 g ( y)dy  c1 dy  c2

82
Borrador del Módulo de Matemática II

d3y d3y  d 2 y´  dy´  2 


En forma similar  g ( y )  3  y´ y´ 2    
dx 3 dx  d y  dy  

3 CASO: Las ecuaciones de la forma F ( x, y , y ,......., y )  0  3 , donde la


k k 1 n

ecuación 3 , no contiene a y , se puede rebajar el orden de la ecuación tomando


como nueva función incógnita la derivada de orden inferior de la ecuación dada a
z  y k , obteniéndose la ecuación F ( x, z, z´,........, z nk )  0.

EJERCICIOS DE APLICACIÓN
Resolver las siguientes ecuaciones diferenciales.
d3y
1.  xe x
dx 3

d 3 y  xe x dx 3   d 3 y   xe x dx 3

d 2 y  ( xe x  e x  c1 )dx 2  d y   xe x dx 2   e x dx 2   c1dx 2
2

dy  xe x  e x  e x  c1 x  c2   dy   xe dx  2 e x dx   c1 xdx   c2 dx
x

c1 x 2
Sol: y  xe  e  2e 
x x x
 c 2 x  c3
2
d2y
2.  ay
dx 2

dy´ y´2 ay 2
y´  ay  y´dy´aydy  0   y´dy´  aydy  c    c1
dy 2 2

dy dx 1
y´ 2c1  ay 2   2c1  ay 2  
dx dy 2c1  ay 2

 
 
arsen c
dy 1 dy 1 y
x
2c1  ay 2
 c2  x  1  2c1
 c2  x 
a 2c1  2
a 2
 y2  
a  a 

 
 
a x  c2   arsen   sen a x  c  
y y
 2c  2
2c1
 1

 a  a

83
Borrador del Módulo de Matemática II

2c1
Sol: y  sen a x  c1 
a

3. xy´´ y´ln 

x

z dz z z


z  y´  xz´ z ln    x  z ln    z ln  dx  xdz  0
 x dx  x  x

z  ux  dz  udx  xdu

 ux 
ux ln  dx  xudx  xdu   0  u ln udx  udx  xdu  0
 x

u ln u  u dx  xdu  0  xu ln u  1

 c  ln x  ln ln u  1  ln k
dx du
 x

u ln u  u

 y´ 
 ln k  x  k ln u  1   1  ln  
x x
ln
ln u  1 k x
x x
1 y´ 1
e k
   e k x   y´
x
x x
1 1
Sol: y  xe k
k e 2 k
 c2

4. xy´yy´´ y´   yy´  x y
2 2 4 3

y  e  y´ z ( x)e   y´´ z´(x)e   z 2 ( x)e 


z ( x ) dx z ( x ) dx z ( x ) dx z ( x ) dx

xze 
zdx
  
e  zdxe  zdx z´ z 2  z 2 e 2  zdx   z 2 e 3 zdx  x 4 e 3 zdx


xze  z´ z 2 e   x 4 e 
3 zdx 3 zdx 3 zdx

xzz´ z 2  x 4

z2
zz´  x3
x

2z 2
2 zz´  2x 3
x
u  z 2  u´ 2 zz´

84
Borrador del Módulo de Matemática II

2u
u´  2 x 3  E.D.L.1er .Orden
x

   2xdx
  
  
2 x 3 dx  c   u  e 2 ln x  e 2 ln x 2 x 3 dx  c  u  x 2  2 xdx  c 
2 dx

ue  e
x

 

 
u  x 2 x 2  c  u  x 4  c  como  y  e 
zdx
 ln y   zdx 

y
z

z 2  x 4  cx 2

y´2
2
 x 4  cx 2  y´ xy x 2  c
y

dy dy
( x x 2  c )dx   0   ( x x 2  c )dx   0
y y

Sol:
3

1 2
x  c  3  ln y  0
1

5.- x 2 yy´´  y  xy´2 sug : y'  yz

y´ yz  y´´ y´z  z´ y

x 2 y y´z  z´ y    y  xyz 
2

   
x 2 yy´z  z´ y 2  y 2  2 xyz  x 2 y 2 z 2  x 2 y 2 z 2  x 2 z´ y 2  y 2  2 xy 2 z  x 2 y 2 z 2

x 2 y 2 z´2 xy 2 z  y 2

 
2 dx
  2xdx  1   2  2  1  
ze  e  2 dx  c   z  e ln x  e ln x  2 dx  c
x

 x    x  

1  2 1  
dx  c   z  2 x  c  z   2
1 1 c
x 2   x 2
z x 
  x x x

y´ x  c dy x  c xc dy dx c dy
 2   2 y  2 dx   0     2 dx   0
y x dx x x y x x y

c x c
ln x   ln y  0 Sol: ln 
x y x
NOTA: Cuando la ecuación diferencial es homogénea para la función y sus

derivadas, la sustitución y´ yz o y  e 


zdx
, reduce el orden de la ecuación
diferencial en una unidad.

5. 4 x yy´ 9 xy  6 x  54 y  108 y  72 y  16
2 2 6 4 2

85
Borrador del Módulo de Matemática II

  
4 x 2 yy´ 3x 3 y 2  2  2 27 y 6  54 y 4  36 y 2  8 
  
4 x 2 yy´ 3x 3 y 2  2  2 3 y 2  2  3


z  3 y 2  2  z´ 6 yy´ y´ y 
6
2 2 3
x z´ 3xz  2 z 3   x 2
3 2
9
z´ z  3z 3 x 2  E.D.BERNOULLI
2x
9 2 6
 2 z 2 z´ z  2
2x x
9 6
w  z 2  w´ 2 z 3 z´  w´ w   2
x x

 
9 dx
  9 dx
x 
6   9  9  6  
we x
e   2 dx  c   w  e ln x e ln x   dx  c 
  x     x  

w
1
x 9
 c
 6 x 7 dx  c  w  9 
x

3
4x

1 1 1 4c  3x 3
w   
z 2
3y  2
2
 2
3 y 2
2 
2
4x8

Sol: 3 y 2  2 
2 4x 9
4c  3x 8
3.13. ECUACIONES DIFERENCIALES LINEALES COEFICIENTES
CONSTANTES
Las ecuaciones diferenciales homogéneas de coeficientes constantes son de la
forma an y n   an1 y n1  .........  a1 y´a0 y  0  1 con ai , i  0,1..............n.

Para resolver estas ecuaciones diferenciales, primero se considera el polinomio


característico de la forma P(r )  an r n  an1r n1  ..........  a1r  a0  0 .

El polinomio P(r ) tiene raíces r1 , r2 ,.........., rn , que pueden ser reales distintas

redes de multiplicidad o complejas.


Para la solución de la ecuación 1 consideramos los siguientes casos:

86
Borrador del Módulo de Matemática II

Caso 1: P(r )  0 , tenga raíces reales distintas r1, r2 ,.r3 ,......., rn entonces el

sistema fundamental de la solución de la ecuación 1 tiene la siguiente forma

e r1x , e r2 x , e r3 x ,.........., e rn x y la solución general es y g  c1e r1x  c2 e r2 x  .........  e rn k .

Caso 2: P(r )  0 , tenga raíces con multiplicidad r1  r2  r3  ...........  rk  r, donde

r es la raíz de multiplicidad k  y n  k  son las demás raíces distintas, entonces

el sistema fundamental de soluciones tiene la forma


e rx , xe rx , x 2 e rx , x k 1e rx , e rk 1x ,............, e rn x y la solución general de la ecuación 1 es:

y g  c1e rx  xc 2 e rx  c3 x 2 e rx  ............  ck x k 1e rx  ck 1e rk 1x  ..........  cn e rn x

Caso3: P(r )  0 tenga raíces complejas r1  1  1i, r2  1  1i, r3   2   2i, r4   2   2i y

las demás raíces reales distintas, entonces el sistema fundamental de solución es


e1x cos 1 x, e1x sen1 x, e 2 x cos  2 x, e 2 x sen 2 x, e r3 x ,............, e rnk y la solución general de
la ecuación 1 es:

y g  c1e1x cos 1 x  c2 e1x sen1 x  c3 e3 x cos  2 x  c4 e 4 x sen 2 x  c5 e r5 x  c6 e r6 x  ...  cn e rn k

EJEMPLOS
Resolver las siguientes ecuaciones diferenciales
1. y´´ y  0

El polinomio característico asociado a la ecuación diferencial es:


P( r )  r 2  1

r 2  1  0  r  1r  1  r1  1, r2  1

Sistema fundamental soluciones : e  x , e x


x
Solucion general : y g  c1e  c2 e
x

2. y´´3 y´2 y  0

P(r )  r 2  3r  2

r 2  3r  2  0  r1  1, r2  2

Sistema fundamental solución : e x , e 2 x

Solución general : y g  c1e  c2 e


x 2x

3. y´´4 y´4 y  0

P(r )  r 2  4r  4

87
Borrador del Módulo de Matemática II

r 2  4r  4  0
r  22  0
r2 raiz multiplici dad 2

Sistema fundamental solución : e 2 x , xe 2 x

Solución general y g  c1e  c2 e


2x 2x

z  y´ z´ y´´

yz´2 yz ln y  z 2  0

z´2 z ln y  z 2  E.D.BERNOULLI

z 2 z´2 z 1 ln y  1   1   z 2 z´2 z 1 ln y  1

 y   y 
dy dy

u  z 1  u´  z 2 z´  u´
u
 2 ln y  u  e e  2 ln y dy  c
y  

u  e ln y e  ln y  2 ln y dy  c  
 u  y ln 2 y  c1 
 ln y 
Sol: ln y  c1tg x  c2 
dy
 yln 2

y  c1 y
 arctg 
 c1 
  x  c2

4. y´´ y´ y  0

P( r )  r 2  r  1

r 2  r 1  0

 1  1  411
r
21

1 3 1 3
r1    i r2    i
2 2 2 2
1 1
 x 3  x 3
2
Sistema fundamental solución : e cos x, xe 2 sen x
2 2
1 1
 x 3  x 3
Solución general y g  c1e 2
cos x  c2 e 2 sen x
2 2
5. y´´´ y´´ y´ y  0

P( r )  r 3  r 2  r  1
r 3  r 2  r 1  0
r 2 r  1  r  1  0

88
Borrador del Módulo de Matemática II

r  1r 2  1  0
r1  1 r2  i  0 r3  i  0

Sistema fundamental solución : e x , e 0 cos x, e 0 senx

Solución general y g  c1e  c2 cos x  c3 senx


x

6. y 4   8 y 2   16 y  0

P(r )  r 4  8r 2  16

r 4  8r 2  16  0

r 2
2
4 0
r  2
r1  2  multiplici dad 2

r2  2  multiplici dad 2

Sistema fundamental solución : e 2 x , xe 2 x , e 2 x , xe 2 x


2 x 2 x
Solución general y g  c1e  c2 xe  c3 e  c4 e
2x 2x

7. y 6   6 y 4   9 y 2   4 y  0

P(r )  r 6  6r 4  9r 2  4

r 6  6r 4  9r 2  4  0

r 2
 2
 4 r 2 1  0
r1  i  multiplici dad 2 r2  i  multiplici dad 2

r3  2i  multiplici dad 2 r4  2i  multiplici dad 2

Sistema fundamental sol : e 0 x cos x, e 0 x x cos x, e 0 x sen.x, e 0 x xsen.x, e 0 x cos 2 x, e 0 x sen2 x

Solución general y g  c1 cos x  c2 x cos x  c3 sen.x  c4 xsen.x  c5 cos 2 x  c6 sen2 x

3.14. SISTEMAS DE ECUACIONES DIFERENCIALES CON COEFICIENTES


CONSTANTES
Un sistema de n ecuaciones diferenciales lineales de primer orden en las
funciones incógnitas x1  1 t , x2   2 t , x3   3 t ,........................, xn   n t  , es de

la forma:

89
Borrador del Módulo de Matemática II

 dx1
 dt  f1 t , x1 , x 2 ,................., x n 
 dx
 2  f 2 t , x1 , x 2 ,..................x n 
 dt
 .

 .
 dx n
 dt  f n t , x1 , x 2 ,.....................x n 

La metodología para resolver un sistema lo explicaremos con los siguientes


ejemplos:
EJERCICIOS DE APLICACIÓN
 dx
  3  2 y  1
1 dx 
1.  dt y  3    3
  2 x  2t  2
dy 2 dt 
 dt

Reemplazar 3 en 2
d  3 dx  1 d 2x 1
    2 x  2t    2 x  2t   x´´2 x  2t
dt  2 2dt  2 dt 2
2
x´´4 x  4t  *  E.D.L.N.H .C.C
-Primero hallamos la solución de la Ecuación Diferencial Lineal homogénea de
Coeficientes Constantes.
x´´4 x  0
Pr   r 2  4

r2  4  0
r1  2i r2  2i

Solución general x g  c1 cos 2 x  c2 sen.x

sol. f : x  x g  x p  solucion.E.D.N.H .C.C

-Para hallar la solución particular utilizamos las reglas establecidas dentro de las
ecuaciones diferenciales No homogéneas de Coeficientes Constantes para lo cual
el caso particular de nuestro ejercicio la solución particular es de la forma:
x p  At  B derivando y sustituyendo en la ecuación dada * , hallar los valores de

A y B.
 x´ p  A
  sustituyo en : *
 x´´ p  B

90
Borrador del Módulo de Matemática II

0  4 At  B  4t
4 At  4B  4t
4 A  4 A 1
   xp  t
B0 B0
La solución de la ecuación * es:

x  xg  x p x  c1 cos 2 x  c2 sen2 x  t

1 dx 
 3    y  3  1  y  1
1
y
2 dt  2

 x  c1 cos 2 x  c2 sen2 x  t
Sol: 
 y 1

 dx
 dt  x  2 y  1 1 dx 
2.  y  x    3
  x  3 y  2
dy 2 dt 
 dt

Reemplazar 3 en 2
d 1  dx  1 d 2x 3 dx 
  x     x  3 y    x  x  
dt  2  2dt  2 dt 2
2 dt 
1 5 3
 x´´ x  x´  x´´3x´5x  0
2 2 2

3  9  20
P(r )  r 2  3r  5 r 2  3r  5  0 r
21

3 11 3 11
r1   i r2   i
2 2 2 2
3 3
11 11
en  2
x x
x g  c1e 2 cos x  c2 e 2 sen x
2 2
 3
x 11
3
x 11
 x  c1e cos x  c2 e 2 sen
2
 x
Sol:  2 2
 x
 y
 2

 dx
 dt  3x  2  3t  2  2  1
y 2 t 3
 t 3 dx 
3.  y  2 3x  3t 2      3
 2 y  2t  1  2
dy  2 2 dt 

 dt

Reemplazar 3 en 2

91
Borrador del Módulo de Matemática II

d  t dx 3   t 3 dx 
2 3x  3t 2      4 3x  3t 2      2t  1
dt  2 dt 2   2 2 dt 

dx d 2x dx
6  12t  1  2 2  12 x  12t 2  2  6  4  2t  1
dt dt dt
x´´5x´6 x  6t 2  4t  3  E.D.L.N.H .C.C
Sol : x  x g  x p

Hallar la solución general haciendo:


x´´5x´6 x  0  *

P(r )  r 2  5r  6 r 2  5r  6  0 r  3r  2  0 r1  3 r2

x g  c1e 2 x  c2 e 3 x

Según las reglas de las ecuaciones lineales homogéneas de coeficientes


constantes, la solución particular es de la forma:
x p  At 2  Bt  C x´ p  2 At  B x' ' p  2 A

2 A  10 At  5B  6 At 2  6Bt  6C  6t 2  4t  3
6A  6  A  1
 10 A  6B  4  B  1
2 A  5B  6C  3
2  5  6C  3  C  0
x g  c1e 2 x  c2 e 3 x  t 2  t

 t 3 
y  2 3x  3t 2    2t  t 
 2 2 

 x  c1e 2 x  c2 e 3 x  t 2  t
Sol: 
 y  6 x  6t  7t  3
2

92

También podría gustarte